37
Red Sirug Page 1 AUDITING THEORY Comprehensive Reviewer Preliminary Grading Period SET 1 1. An independent financial statement audit is important to financial statements users because it: a. Objectively examines and reports on special-purpose financial statements. b. It reduces cost of capital c. Objectively examines and reports on general-purpose financial statements. d. Objectively reports on the accuracy of information in the financial statements. 2. In relation to auditing, which of the following is a correct phrase? a. Auditing communicates results to management. b. Auditing involves obtaining evidence regarding action and events. c. Auditing evaluates assertions regarding evidence. d. Auditing subjectively obtains and evaluates evidence. 3. Which of the following is not an output of an independent audit engagement? a. Management letter. b. Audit report. c. Engagement letter. d. Audited financial statements. 4. The best description of the auditor’s responsibility with respect to audited financial statement is: a. The auditor's responsibility on fair presentation of financial statements is limited only up to the date of the audit report. b. The auditor is responsible for detecting misstatements on the financial statements. c. The responsibility over the financial statements rests with the management. d. The auditor's responsibility is limited to the expression of opinion on the financial statements. 5. When a CPA expresses an opinion on the financial statements, his responsibilities extend to a. The underlying wisdom of the client's management decision. b. Active participation in the implementation of the advice given to the client. c. An ongoing responsibility for the client's solvency. d. Whether the results of the client's operating decisions are fairly presented in the financial statements. 6. The accuracy of information included in the footnotes that accompany the audited financial statements of a company whose shares are traded on a stock exchange is the primary responsibility of a. The stock exchange officials. b. The company's management. c. The independent auditor. d. The Securities and Exchange Commission. 7. The responsibility for adopting sound accounting policies, maintaining adequate internal control, and making fair representations in the financial statements rests a. With management b. With the independent auditor c. Equally with management and the auditor d. With the internal audit department. 8. Audit standards require an auditor to: a. Perform procedures that are designed to detect all instances of fraud. b. Provide reasonable assurance that the financial statements are not materially misstated. c. Issue an unqualified opinion only when the auditor is satisfied that no instances of fraud have occurred. d. Design the audit program to meet financial statement users' expectations concerning fraud. 9. Generally, the decision to notify parties outside the client's organization regarding an illegal act is the responsibility of the a. Outside legal counsel. b. Independent auditor. c. Management.

MCQs - prelims

  • Upload
    emc2mcv

  • View
    576

  • Download
    8

Embed Size (px)

DESCRIPTION

Auditing

Citation preview

Page 1: MCQs - prelims

Red Sirug Page 1

AUDITING THEORY

Comprehensive Reviewer

Preliminary Grading Period

SET 1

1. An independent financial statement audit is important to financial statements users because it:

a. Objectively examines and reports on special-purpose financial statements.

b. It reduces cost of capital

c. Objectively examines and reports on general-purpose financial statements.

d. Objectively reports on the accuracy of information in the financial statements.

2. In relation to auditing, which of the following is a correct phrase?

a. Auditing communicates results to management.

b. Audit ing involves obtaining evidence regarding action and events.

c. Auditing evaluates assertions regarding evidence.

d. Auditing subjectively obtains and evaluates evidence.

3. Which of the following is not an output of an independent audit engagement?

a. Management letter.

b. Audit report.

c. Engagement letter.

d. Audited financial statements.

4. The best description of the auditor’s responsibility with respect to audited financial statement is:

a. The auditor's responsibility on fair presentation of financial statements is limited only up to

the date of the audit report.

b. The auditor is responsible for detecting misstatements on the financial statements.

c. The responsibility over the financial statements rests with the management.

d. The auditor's responsibility is limited to the expression of opinion on the financial

statements.

5. When a CPA expresses an opinion on the financial statements, his responsibilities extend to

a. The underlying wisdom of the client's management decision.

b. Active participation in the implementation of the advice given to the client.

c. An ongoing responsibility for the client's solvency.

d. Whether the results of the client's operating decisions are fairly presented in the financial

statements.

6. The accuracy of information included in the footnotes that accompany the audited financial statements of a

company whose shares are traded on a stock exchange is the primary responsibility of

a. The stock exchange officials.

b. The company's management.

c. The independent auditor.

d. The Securities and Exchange Commission.

7. The responsibility for adopting sound accounting policies, maintaining adequate internal control, and

making fair representations in the financial statements rests

a. With management

b. With the independent auditor

c. Equally with management and the auditor

d. With the internal audit department.

8. Audit standards require an auditor to:

a. Perform procedures that are designed to detect all instances of fraud.

b. Provide reasonable assurance that the financial statements are not materially misstated.

c. Issue an unqualified opinion only when the auditor is satisfied that no instances of fraud have

occurred.

d. Design the audit program to meet financial statement users' expectations concerning fraud.

9. Generally, the decision to notify parties outside the client's organization regarding an illegal act is the

responsibility of the

a. Outside legal counsel.

b. Independent auditor.

c. Management.

Page 2: MCQs - prelims

Red Sirug Page 2

d. Internal auditors.

10. If requested to perform a review engagement for a nonpublic entity in which an accountant has an

immaterial direct financial interest, the accountant is

a. Independent because the financial interest is immaterial and, therefore, may issue a review report.

b. Not independent and, therefore, may not issue a review report.

c. Not independent and, therefore, may not be associated with the financial statements.

d. Not independent and, therefore, may issue a review report, but may not issue an auditor's opinion.

11. Solicitation consists of the various means that CPA firms use to engage new clients. Which one-of the

following would not be an example of solicitation?

a. Advertisements in the yellow pages of a phone book.

b. Accepting new clients that approach the firm.

c. Taking prospective clients to lunch.

d. Offering seminars on current tax law changes to potential clients.

12. Which of the following activities is not prohibited for the CPA firm's attestation service clients?

a. Referral fees on audit jobs.

b. Competitive bidding on audit jobs.

c. Contingent fees on audit jobs.

d. Commissions for obtaining client services on audit jobs.

13. Family and personal relationships between a member of the assurance team and a director, an officer or

certain employees, depending on their role, of the assurance client, least likely create

a. Self-review threat.

b. Self-interest threat.

c. Intimidation threat.

d. Familiarity threat.

14. A director, an officer or an employee of the assurance client in a position to exert direct and significant

influence over the subject matter of the assurance engagement has been a member of the assurance team

or partner of the firm. This situation least likely create

a. Self-interest threat.

b. Advocacy threat.

c. Intimidation threat.

d. Familiarity threat.

15. A former officer, director or employee of the assurance client serves as a member of the assurance team.

This situation will least likely create

a. Self-interest threat.

b. Self-review threat.

c. Familiarity threat.

d. Int imidation threat.

16. Which of the following will least likely impair independence?

a. A partner or employee of the firm serves as an officer or as a director on the board of an assurance

client.

b. An immediate family member of a member of the assurance team is a director, an officer or an

employee of the assurance client in a position to exert direct and significant influence over the subject

matter of the assurance engagement.

c. A partner or employee of the firm or a network f irm serves as Company Secretary for an

audit client, the dut ies and functions undertaken are limited to those of a rout ine and

formal administrative nature as such as the preparation of minutes and maintenance of

statutory returns.

d. A member of the assurance team participates in the assurance engagement while knowing, or having

reason to believe, that he or she is to, or may, join the assurance client some time in the future.

17. When a CPA firm is requested to provide a written or oral opinion on the application of accounting

principles or the type of audit opinion that would be issued for a specific or hypothetical transaction

relating to an audit client of another CPA firm, primary among the requirements set forth is that

a. Client is entitled to confidentiality, so the consulting CPA firm is forbidden form communicating with

the CPA firm which does the audit.

b. Client is not entitled to confidentiality under these circumstances, so the existing auditors should share

all information with the consulting CPA firm.

c. The consulted CPA firm should communicate with the entity's existing auditors to

ascertain all the available facts relevant to forming a professional judgment on the

Page 3: MCQs - prelims

Red Sirug Page 3

matters the firm has been requested to report on.

d. Client is entitled to confidentiality, so the CPA firm which does audit should refuse to share any

information with the consulting CPA firm under any circumstances.

18. A professional accountant has a professional duty or right to disclose confidential information in each of

the following, except:

a. To disclose to BIR fraudulent scheme committed by the client on payment of income tax.

b. To comply with technical standards and ethics requirements.

c. To comply with the quality review of a member body or professional body

d. To respond to an inquiry or investigation by a member body or regulatory body.

19. Which of the following is not likely a threat to independence?

a. Acting as an advocate on behalf of an assurance client in litigation or in resolving disputes with third

parties.

b. Long association of a senior member of the assurance team with the assurance client.

c. Threat of replacement over a disagreement with the application of an accounting principle.

d. Owning immaterial indirect financial interest in an audit client.

20. When threats to independence that are not clearly insignificant are identified, the following are

appropriate, except:

a. When the firm decides to accept or continue the assurance engagement, the decision need

not be documented provided the threats ident ified were eliminated.

b. Professional judgment is used to determine the appropriate safeguards to eliminate threats to

independence or to reduce them to an acceptable level.

c. In situations when no safeguards are available to reduce the threat to an acceptable level, the only

possible actions are to eliminate the activities or interest creating the threat, or to refuse to accept or

continue the assurance engagement.

d. The evaluation of the significance of any threats to independence and the safeguards necessary to

reduce any threats to an acceptable level, takes into account the public interest.

21. The following loans and guarantees would not create a threat to independence, except:

a. A loan from, or a guarantee thereof by, an assurance client that is a bank or a similar institution, to

the firm, provided the loan is made under normal lending procedures, terms and requirements and

the loan is immaterial to both the firm and the assurance client.

b. A loan from, or a guarantee thereof by, an assurance client that is a bank or a similar institution, to a

member of the assurance team or their immediate family, provided the loan is made under normal

lending procedures, terms and requirements.

c. If the firm, or a member of the assurance team, makes a loan to an assurance client that

is not a bank or similar institut ion, or guarantees such an assurance client's borrowing.

d. Deposits made by, or brokerage accounts of, a firm or a member of the assurance team with an

assurance client that is a bank, broker or similar institution, provided the deposit or account is held

under normal commercial terms.

22. Examples of circumstances that may create self-interest threat include:

a. Contingent fees relating to assurance engagements.

b. A direct financial interest or material indirect financial interest in an assurance client.

c. A loan or guarantee to or from an assurance client or any of its directors or officers.

d. All of the above

23. Examples of circumstances that may create self-review threat least likely include

a. Potential employment with an assurance client.

b. Preparation of original data used to generate financial statements or preparation of other records that

are the subject matter of the assurance engagement.

c. A member of the assurance team being, or having recently been, an employee of the assurance client

in a position to exert direct and significant influence over the subject matter of the assurance

engagement.

d. Performing services for an assurance client that directly affect the subject matter of the assurance

engagement.

24. A CPA-lawyer, acting as a legal counsel to one of his audit client, is an example of

a. Advocacy threat

b. Familiarity threat

c. Self-interest threat

d. Self-review threat

25. Examples of circumstances that may create familiarity threat least likely include

Page 4: MCQs - prelims

Red Sirug Page 4

a. A former partner of the firm being a director, officer of the assurance client or an employee in a

position to exert direct and significant influence over the subject matter of the assurance engagement.

b. Dealing in, or being a promoter of, share or other securities in an assurance client.

c. A member of the assurance team having an immediate family member or close family member who is

a director or officer of the assurance client.

d. A member of the assurance team having an immediate family member or close family member who, as

an employee of the assurance client, is in a position to exert direct and significant influence over the

subject matter of the assurance engagement.

26. The following are modifications made to the IFAC Code to consider Philippine regulatory requirements and

circumstances, except

a. The period for rotation of the lead engagement partner was cha nged from five to seven

years.

b. Advertising and solicitation by individual professional accountants in public practice were not

permitted in the Philippines.

c. Additional examples relating to anniversaries and websites

wherein publicity is acceptable, as provided in boa resolution 19, series of 2000, were included.

d. Payment and receipt of commissions were not permitted in the Philippines.

27. The Code of Professional Ethics states, in part, that a CPA should maintain integrity and objectivity.

Objectivity refers to the CPA's ability to

a. Insist on all matters regarding audit procedures.

b. Determine the materiality of items.

c. Determine accounting practices that were consistently applied.

d. Maintain an impartial attitude on all matters which come under his review.

28. The network firms are required to be independent of the client

a. For assurance engagements provided to an audit client.

b. For assurance engagements provided to clients that are not audit clients, when the report is not

expressly restricted for use by identified users.

c. For assurance engagements provided to clients that are not audit clients, when the assurance report is

expressly restricted for use by identified users.

d. All of the above

29. Which of the following is incorrect regarding independence?

a. Independence consists of independence of mind and independence in appearance.

b. Independence is a combination of impart iality, intellectual honesty and a freedom from

conflicts of interest.

c. Independence of mind is the state of mind that permits the provision of an opinion without being

affected by influences that compromise professional judgment, allowing an individual to act with

integrity, and exercise objectivity and professional skepticism.

d. Independence in appearance is the avoidance of facts and circumstances that are so significant a

reasonable and informed third party, having knowledge of all relevant information, including any

safeguards applied, would reasonably conclude a firm's or a member of the assurance team's integrity,

objectivity or professional skepticism had been compromised.

30. A CPA firm is considered independent when it performs which of the following services for a publicly-traded

audit client?

a. Serving as a member of the client's board of directors.

b. Determining which accounting policies will be adopted by the

client.

c. Accounting information system design and implementation.

d. Tax return preparation as approved by the board of directors.

31. In connection with the examination of financial statements, an independent auditor could be responsible

for failure to detect a material fraud if

a. Statistical sampling techniques were not used on the audit engagement.

b. The fraud was perpetrated by one client employee, who circumvented the existing internal control.

c. The auditor planned the work in a hasty and inefficient manner.

d. Accountants performing important parts of the work failed to discover a close relationship between the

treasurer and the cashier.

32. Which of the following conditions suggests auditor negligence?

a. Failure to detect collusive fraud perpetrated by members of middle management.

b. Failure to detect collusive fraud perpetrated by members of top management.

c. Failure to detect errors occurring outside the internal control structure.

Page 5: MCQs - prelims

Red Sirug Page 5

d. Failure to detect material errors under condit ions of weak internal control.

33. Conflict between financial statement users and auditors often arises because of the

a. High cost of performing an audit.

b. Expectation gap.

c. Technical vocabulary which the auditor uses in the report.

d. Placement of the auditor's report in the back of the client's annual report where it is hard to locate.

34. Which of the following is best considered a fraud?

a. Inability to provide due diligence.

b. Intent ional misrepresentation of financial information.

c. Declining to finish work on client in light of a valid contract.

d. Not acting professionally while performing services.

35. Anyone identified to the auditor by name prior to the audit who is to be the principal recipient of the

auditor's report is a

a. Primary beneficiary.

b. Third party.

c. Foreseen beneficiary.

d. Secondary beneficiary.

36. Quality control procedures are applicable to the individual audit engagement. The implementation of such

quality control procedures is responsibility of the:

a. CPA firm.

b. Engagement team.

c. Quality control reviewer.

d. Manager assigned to the engagement.

37. Which of the following statements is incorrect?

a. In an audit, the independent auditor attempts to corroborate assertions made by the company’s

management in connection with each account, class of transactions, and disclosures found in a set of

financial statements.

b. Because of the attest function, financial statements are the responsibility of the

independent auditor.

c. The term “materiality” refers to any factor of a size or type that would impact an outside decision-

maker’s decision about a set of financial statements.

d. The role of the independent auditor is to gain sufficient appropriate evidence so as to provide

reasonable assurance that material misstatements do not exist in any of the assertions made by

management.

38. Which of the following statements is correct?

a. Sufficiency refers to the quality of evidence, while appropriateness refers to the quantity of evidence.

b. The reliability of evidence is influenced not by its nature but by its source.

c. The performance of consult ing services for audit clients does not, in and of itself, impair

the auditor’s independence.

d. A belief that management and those charged with governance are honest and have integrity relieves

the auditor of the need to maintain professional scepticism.

39. Which of the following statements is correct?

a. The fair presentation of audited financial statements in accordance with applicable financial reporting

framework is an implicit part of the auditor’s responsibility.

b. Professional judgment can be used as the justification for the decisions made by the auditor that are

not otherwise supported by the facts and circumstances of the engagement or sufficient appropriate

evidence.

c. Appropriateness is the measure of the quality of evidence, that is, its reliability and persuasiveness.

d. Most CPAs, including those who provide audit and tax services, also provide consult ing

services to their clients.

40. Individual CPAs, Firms or Partnerships of CPAs, including partners and staff members thereof shall register

with the BOA and the PRC. Assuming that the application for registration of Sisip and Co., CPAs was

approved on August 30, 2005, which of the following is true?

a. The registration will expire on Dec. 31, 2007.

b. The registration must be renewed on September 30, 2007.

c. The registration will expire on August 30, 2007 since the validity of the certificate of registration is

three years.

d. The registration will expire on Dec. 31, 2007 which is also the last day of renewal of certificate of

Page 6: MCQs - prelims

Red Sirug Page 6

registration.

41. Which of the following statements regarding RA 9298 is true?

a. A person shall be considered to be in the professional practice of accounting if, as an

officer in a private enterprise, he makes decisions requiring professional accounting

knowledge.

b. The Professional Regulation Commission has the authority to remove any member of the Board of

Accountancy for negligence, incompetence, or any other just cause.

c. Insanity is not a ground for proceeding against a CPA.

d. After three years, subject to certain conditions, the Board of Accountancy may order the reinstatement

of a CPA whose certificate of registration has been revoked.

42. Auditor’s responsibility for detecting noncompliance is limited to those:

a. Direct-effect noncompliance.

b. Material direct-effect noncompliance.

c. Material indirect-effect noncompliance.

d. All noncompliance since they affect the financial sta tements directly or indirectly.

43. Auditors would perform the following steps in which order?

a. Determine audit risk; assess control risk; determine detection risk; set materiality.

b. Set materiality; assess control risk; determine detection risk; determine audi t risk.

c. Set materiality; determine audit risk; assess control risk; determine detection risk.

d. Determine audit risk; set materiality; assess control risk; determine detection risk.

44. Which of the following statements is an not an example of an inherent limitation of internal control?

a. The effectiveness of control procedures depends on segregation of dut ies.

b. Errors may arise from mistakes in judgments.

c. Most internal controls tend to be directed at routine transactions rather than non-routine transactions.

d. The cost-benefit relationship is a primary criterion in designing internal control,

45. The confidential relationship applies to:

a. Only audit and attestation services.

b. Only audit and attestation services since these types of services also require independence.

c. Audit and MAS services, but not compilation engagements.

d. All services provided by CPAs.

46. Compliance with the independence requirement is necessary whenever a CPA performs:

a. Non-assurance services

b. Professional services

c. Tax consultancy services

d. Assurance services

47. Which of the following is least likely an advantage to the client and the auditor as a result of early

appointment of the auditor?

a. The audit will be completed expeditiously.

b. The auditor will be able to perform the examination more efficiently and will be finished at an early

date after the year-end.

c. The auditor will be able to complete the audit work in less time.

d. Potential problems are identified and resolved on a timely basis.

48. Which of the following procedures would an auditor be most likely to perform in planning a financial

statement audit?

a. Performing analytical procedures to identify material fraud and material error.

b. Performing analytical procedures to ident ify areas that may represent specific risks.

c. Obtaining a written representation letter from the client to emphasize management's responsibilities.

d. Reading the minutes of stockholder and director meetings to discover whether any unusual

transactions have occurred.

49. The primary purpose why an auditor obtains an understanding of the ent ity and its environment is to:

a. Develop an attitude of professional skepticism concerning management's financial statement

assertions.

b. Make constructive suggestions concerning improvements to the client's internal control.

c. Understand the events and transactions that may have an effect on the client's financial

statements.

d. Evaluate whether the aggregation of known misstatements causes the financial statements taken as a

whole to be materially misstated.

Page 7: MCQs - prelims

Red Sirug Page 7

50. Auditors allocate the preliminary judgment about materiality to account balances. Such preliminary

materiality is known as:

a. Tolerable error.

b. Planning materiality.

c. Tolerable misstatement.

d. Tolerable materiality.

51. Although generalized audit programs may be used, the auditor should design an audit program suitable to

the circumstances on particular engagements. A complete audit program for an engagement generally

should be developed

a. Prior to beginning the actual audit work.

b. Whenever the auditor has completed an evaluat ion of the relevant internal control.

c. After completing the study of internal control.

d. After reviewing the client's accounting records and procedures.

52. Which of the following is least likely a potential effect of an auditor's decision to reduce acceptable audit

risk at low level?

a. Special care is required in assigning experienced staff.

b. Review of the working papers by personnel who were not assigned to the engagement.

c. More evidence is required.

d. Less evidence is required.

53. Which of the following statements is true?

a. It is usually equally difficult for the auditor to uncover errors or fraud.

b. It is usually easier for the auditor to uncover errors than fraud.

c. Usually, the auditor does not design procedures to uncover fraud or errors.

d. It is usually easier for the auditor to uncover management fraud than employee fraud.

54. The risk of not detecting material misstatement resulting from fraud is greater than the risk of not

detecting a material misstatement arising from error, because:

a. Fraud ordinarily involves acts designed to conceal it.

b. The auditor designs only procedures to detect material error but no procedures are designed to detect

material fraud.

c. The PSA do not require the auditor to discover information that is indicative of fraud.

d. It is the responsibility of the management to detect fraud and the auditor's responsibility is confined

only to the detection of material errors.

55. Which of the following statements about noncompliance is incorrect?

a. An audit in accordance with PSA can not be expected to detect noncompliance with all laws and

regulations.

b. The determination as to whether a part icular act constitutes noncompliance is ult imately

based on the judgment of the auditor.

c. An auditor can not be held responsible for preventing noncompliance.

d. It is management's responsibility to ensure that entity's operations are conducted in accordance with

laws and regulations.

56. Which of the following statements best describes why the auditor's examination cannot reasonably be

expected detect all acts of noncompliance with existing laws and regulations?

a. Noncompliance may be perpetrated by the only person in the client's organization with access to both

assets and the accounting records.

b. Noncompliance may involve conduct designed to conceal it, such as collusion, forgery,

deliberate failure to record transactions.

c. Acts of noncompliance by clients often relates to accounting aspects rather than operating aspects.

d. The client's internal control may be so strong that the auditor performs only minimal substantive

testing.

57. Presented below are circumstances that may indicate the occurrence of nonco mpliance with laws and

regulations, except:

a. Payment of fines or penalties

b. Payment for unspecified services to consultants, related parties, or government employees.

c. Purchasing at prices significant ly above or below book value.

d. Purchasing at prices significantly above or below market price.

58. Which of the following is not an information source for developing analytical procedures used in the audit?

a. Key relationships among financial statement elements.

Page 8: MCQs - prelims

Red Sirug Page 8

b. Relationships between financial and relevant nonfinancial data.

c. Comparison of current year financial data with project ions for next year's financial results

d. Comparison of financial data with anticipated results.

59. It is acceptable for the auditor to prepare:

a. The financial statements for the client.

b. The notes to financial statements for the client.

c. A draft of the financial statements for the client.

d. A draft of the financial statements and notes to the financial statements for the client.

60. The assessment of the risks of material misstatement at the financial statement level is affected by the

auditor’s understanding of the control environment. Weaknesses in the control environment ordinarily will

lead the auditor to

a. Modify the nature of audit procedures to obtain more persuasive audit evidence.

b. Have more confidence in internal control and the reliability of audit evidence generated internally within

the entity.

c. Conduct some audit procedures at an interim date rather than at period end.

d. Decrease the number of locations to be included in the audit scope.

61. Which of the following is a conceptual difference between the attestation standards and generally accepted

auditing standards?

a. The attestation standards provide a framework for the attest function beyond historical

financial statements.

b. The attestation standards do not permit an attest engagement to be part of a business acquisition

study or a feasibility study.

c. Attestation standards do not require independence in mental attitude.

d. Attestation standards do not include standards of reporting.

62. When an accountant performs more than one level of service, he generally should issue a report that is

appropriate for:

a. The lowest level of service rendered.

b. A compilation engagement.

c. The highest level of service rendered.

d. A review engagement.

63. An accountant who reviews the financial statements should issue a report stating that a review

a. Is substant ially less in scope than an audit.

b. Provides negative assurance that the internal control is functioning as designed.

c. Provides only a limited assurance that the financial statements are fairly presented.

d. Is substantially more in scope than a compilation

64. Which of the following is required to be performed in an audit but not in review engagement?

a. Complying with the "Code of Professional Ethics for Certified Public Accountants" promulgated by the

Board of Accountancy

b. Planning the engagement

c. Agreeing on the terms of engagement

d. Studying and evaluat ing internal control structure

65. Which statement is incorrect regarding procedures and evidence obtained in a review engagement?

a. The auditor should apply judgment in determining the specific nature, timing and extent of review

procedures.

b. The auditor should apply the same materiality considerations as would be applied if an audit opinion

on the financial statements were being given.

c. There is a greater risk that misstatements will not be detected in an audit than in a

review.

d. The judgment as to what is material is made by reference to the information on which the auditor is

reporting and the needs of those relying on that information, not to the level of assurance provided.

66. A CPA is conducting the first examination of a non-public company's financial statements. The CPA hopes

to reduce the audit work by consulting with the predecessor auditor and reviewing the predecessor's

working papers. This procedure is

a. Acceptable if the CPA refers in the audit report to reliance upon the predecessor auditor's work.

b. Required if the CPA is to render an unqualified opinion.

c. Acceptable if the client and the predecessor auditor agree to it.

d. Unacceptable because the CPA should bring an independent viewpoint to a new engagement.

Page 9: MCQs - prelims

Red Sirug Page 9

67. Which of the following would not fit the description of a related-party transaction?

a. Sales of merchandise between a parent company and its subsidiary.

b. An unusually large sale of merchandise to the company's best and largest customer.

c. Exchanges of equipment between two companies owned by the same person.

d. Loans to corporate officers at market rates of interest with a regular repayment schedule.

68. The audit work performed by each assistant should be reviewed to determine whether it was adequately

performed and to evaluate whether the

a. Audit procedures performed are approved in the professional standards.

b. Audit has been performed by persons having adequate technical training and proficiency as auditors.

c. Results are consistent with the conclusions to be presented in the auditor's report.

d. Auditor's system of quality control has been maintained at a high level.

69. Which of the following is not a document or record that should be examined early in the engagement?

a. Corporate charter and by-laws.

b. Contracts.

c. Management letter.

d. Minutes of board of directors' and stockholders' meetings.

70. Philippine Standards on Auditing require auditors to assess the risk of material misstatements due to fraud

a. For first-time audits.

b. Sufficient to find any frauds which may exist.

c. For every audit.

d. Whenever it would be appropriate.

71. A measure of the auditor's assessment of the likelihood that there are material misstatements in a segment

before considering the effectiveness of the internal control structure is

a. Inherent risk.

b. Acceptable audit risk.

c. Statistical risk.

d. Control risk.

72. Which of the following statements best describes the auditor's responsibility to detect material errors and

fraud?

a. The auditor is responsible for the failure to detect material errors and frauds only when such failure

results from the misapplication of generally accepted accounting principles.

b. The auditor is responsible for the failure to detect material errors and frauds only when the auditor

fails to confirm receivables or observe inventories.

c. The audit should be designed to provide reasonable assurance that material errors and

fraud are detected.

d. Extended auditing procedures are required to detect unrecorded transactions even if there is no

evidence that material errors and frauds may exist.

73. In connection with the examination of financial statements, an independent auditor could be responsible

for failure to detect a material fraud if

e. Accountants performing important parts of the work failed to discover a close relationship between the

treasurer and the cashier.

f. The auditor planned the work in a hasty and inefficient manner.

g. Statistical sampling techniques were not used on the audit engagement.

h. The fraud was perpetrated by one client employee, who circumvented the existing internal control.

74. Which of the following is not true regarding planning in an electronic environment?

a. The definition of auditing is not changed

b. The purposes of auditing is not changed

c. The procedures used are not changed

d. Auditing standards are not changed

75. Which of the following statements best identifies the two types of fraud?

a. Theft of assets and employee fraud.

b. Management fraud and employee fraud

c. Misappropriation of asset and defalcation

d. Fraudulent financial reporting and management fraud.

76. Which of the following terms relates to the embezzling of receipts?

a. Misrepresentation

b. Misapplication

Page 10: MCQs - prelims

Red Sirug Page 10

c. Misappropriation

d. Manipulation

77. Which of the following fraud risk factors relate to misstatement arising from fraudule nt financial reporting?

a. Personal financial obligations may create pressure on management or employees with access to cash

or other assets susceptible to theft to misappropriate those assets.

b. Adverse relationships between the entity and employees with access to cash or other assets

susceptible to theft may motivate those employees to misappropriate those assets.

c. Inadequate internal control over assets may increase the susceptibility of misappropriation of those

assets. For example, misappropriation of assets may occur because there is the following:

d. Recurring negative cash flows from operations or an inability to generate cash flows from

operations while reporting earnings and earnings growth.

78. An auditor's examination performed in accordance with generally accepted auditing standards generally

should

a. Be expected to provide assurance that illegal acts will be detected where internal control is effective.

b. Be relied upon to disclose violations of truth-in-lending act.

c. Encompass a plan to search actively for illegalities which relate to operating aspects.

d. Not be relied upon to provide assurance that illegal acts will be detected.

79. Which of the following concepts of materiality is incorrect?

a. Materiality is based on quantitative and non-quantitative factors.

b. Materiality is a matter of professional audit judgment.

c. Materiality does not apply if internal control is highly effective.

d. Materiality is more closely related to the fieldwork and reporting standards than to general standards.

80. Which of the following statements is incorrect about materiality?

a. The concept of materiality recognizes that some matters are important for fair presentation of financial

statements in conformity with GAAP, while other matters are not important.

b. An auditor considers materiality for planning purposes in terms of the largest aggregate

level of misstatements that could be material to any one of the financial statements.

c. Materiality judgments are made in light of surrounding circumstances and necessarily involve both

quantitative and qualitative judgments.

d. An auditor’s consideration of materiality is influenced by the auditor’s perception of the needs of a

reasonable person who will rely on the financial statements.

81. After discovering that a related-party transaction exists, the auditor should be aware that the

a. Transaction is assumed to be outside the ordinary course of business.

b. Substance of the transaction could be significant ly different from its form.

c. Adequacy of disclosure of the transaction is secondary to its legal form.

d. Financial statements should recognize the legal form of the transaction rather than its substance.

82. Auditors focus on

a. Areas where the risk of material errors and irregularities is least.

b. Areas where the risk of material errors and irregularities is greatest.

c. All areas equally.

d. A random selection of all areas.

83. Audit risk components consist of inherent, control and detection risks. Which of them is are dependent

variable(s)?

a. Inherent risk

b. Control risk

c. Detection risk

d. Inherent and control risks

84. The risk that the audit will fail to uncover a material misstatement is eliminated

a. When the auditor has complied with generally accepted auditing standards.

b. If a client has strong internal controls.

c. If a client follows generally accepted accounting principles.

d. Under no circumstances.

85. The probability of an auditor's procedures leading to the conclusion that a material error does not exist in

an account balance when, in fact, such error does exist is referred to as

a. Prevention risk.

b. Inherent risk.

c. Control risk.

Page 11: MCQs - prelims

Red Sirug Page 11

d. Detection risk.

86. Which of the following statements is true with regard to the relationship among audit risk, audit evidence,

and materiality?

a. The lower the inherent risk and control risk, the lower the aggregate materiality threshold.

b. Under condit ions of high inherent and control risk, the auditor should place more

emphasis on obtaining external evidence and should reduce reliance on internal evidence.

c. Where inherent risk is high and control risk is low, the auditor may safely ignore inherent risk.

d. Aggregate materiality thresholds should not change under conditions of changing risk levels.

87. Which of the following does an auditor least likely perform in assessing audit risk?

a. Understand the economic substance of significant transactions completed by the client.

b. Understand the entity and the industry in which it operates.

c. Gather audit evidence in support of recorded transactions.

d. Obtain an understanding of the client's system of internal control.

88. Which of the following is most likely to be an overall response to fraud risks identified in an audit?

a. Supervise members of the audit team less closely and rely more upon judgment.

b. Use less predictable audit procedures.

c. Use only certified public accountants on the engagement.

d. Place increased emphasis on the audit of objective transactions rather than subjective transactions.

89. If the auditor is convinced that the client has an excellent internal control structure, the amount of audit

evidence to be gathered.

a. Can be significant ly less than where internal control is not adequate.

b. Will not be affected since the auditor must arrive at an independently determined opinion.

c. Must be increased to support the auditor's belief.

d. Is not determinable.

90. Why should the auditor plan more work on individual accounts as lower acceptable levels of both audit risk

and materiality are established?

a. To find smaller errors

b. To find larger errors

c. To increase the tolerable error in the accounts

d. To decrease the risk of overreliance

91. Which of the following most accurately summarizes what is meant by the term “material miss tatement?”

a. Fraud and direct-effect illegal acts.

b. Fraud involving senior management and material fraud.

c. Material error, material fraud, and certain illegal acts.

d. Material error and material i llegal acts.

92. The risk of fraudulent financial reporting increases in the presence of

a. Substantial increases in sales.

b. Incentive systems based on operating income.

c. Improved control systems.

d. Frequent changes in suppliers.

93. Which of the following is most likely to be considered a risk factor relating to fraudulent financial reporting?

a. Domination of management by top executives.

b. Negative cash flows from operat ions.

c. Large amounts of cash processed.

d. Small high-dollar inventory items.

94. When planning the audit, if the auditor has no reason to believe that illegal acts e xist, the auditor should

a. Make inquiries of management regarding their policies and their knowledge of violations,

and then rely on normal audit procedures to detect errors, irregularit ies, and illegalit ies.

b. Still include some audit procedures designed specifically to uncover illegalities.

c. Ignore the topic.

d. Include audit procedures which have a strong probability of detecting illegal acts.

95. Which statement best describes the emphasis of the systems and substantive approaches in the audit

plan?

a. A thoroughly designed systems approach to auditing can eliminate the need for any substantive

procedures.

b. The systems approach focuses on detailed testing of specific accounts for accuracy, while the

Page 12: MCQs - prelims

Red Sirug Page 12

substantive approach is the testing controls to make sure they are effective.

c. The systems approach focuses on testing controls to make sure they are effective, while

the substantive approach is the detailed testing of specific accounts for accuracy.

d. The systems approach focuses on the use of computer systems to aid in the audit while the

substantive approach focuses on more manual tests.

96. Audit programs should be designed so that

a. Most of the required procedures can be performed as interim work.

b. The audit evidence gathered supports the auditor’s conclusions.

c. Inherent risk is assessed at a sufficiently low level.

d. The auditor can make constructive suggestions to management.

97. In designing written audit programs, an auditor should establish specific audit objectives that relate

primarily to the

a. Financial statement assertions.

b. Timing of audit procedures.

c. Cost-benefit of gathering evidence.

d. Selected audit techniques.

98. An audit program provides proof that

a. Sufficient appropriate evidence was obtained.

b. The work was adequately planned.

c. There was a proper study and evaluation of internal control.

d. There was compliance with generally accepted standards of reporting.

99. The audit program usually cannot be finalized until the

a. Reportable conditions have been communicated to the audit committee of the board of directors.

b. Engagement letter has been signed by the auditor and the client.

c. Consideration of the entity’s internal control has been completed.

d. Search for unrecorded liabilities has been performed and documented.

100. A person or firm possessing special skill, knowledge and experience in a particular field excluding

accounting and auditing.

a. Quality control reviewer

b. Multiskilled personnel

c. Expert

d. Taxation specialist

SET 2

1. The Code of Ethics for Professional Accountants in the Philippines is applicable to professional services in

the Philippines on or before:

a. December 31, 2009

b. July 1, 2004

c. June 30, 2008

d. June 30, 2009

2. Which part of the Code of Ethics applies to professional accountants in public practice?

a. Part A

b. Part B

c. Part A and Part B

d. Part C

3. This fundamental ethical principle prohibits association of professional accountants with reports, returns,

communications and other information that contains materially false or misleading information or

statements.

a. Integrity

b. Objectivity

c. Professional competence and due care

d. Professional behavior

4. The principle professional competence and due care imposes which of the following obligations on

professional accountants?

a. To maintain professional knowledge and skill at the level required to ensure that a client

or employer receives competent profess ional service.

Page 13: MCQs - prelims

Red Sirug Page 13

b. To comply with relevant laws and regulations and avoid any action that discredits the profession.

c. Not to override or compromise his professional or business judgment because of bias, conflict of

interest or undue influence of others.

d. To be fair and truthful.

5. Attainment of professional competence requires the following, except:

a. Initially, a high standard of education.

b. Specific education, training, and examination in professionally relevant subjects.

c. Whether prescribed or not, a period of work experience.

d. A continuing awareness and an understanding of relevant technical professional and

business developments.

6. A draft of statement, studies or standards should be discussed by the Council en banc. How many

members of the AASC are required to approve the draft for exposure?

a. Eight

b. Ten

c. Twelve

d. Majority

7. How many members of the AASC are needed to approved the exposed draft as PSA

a. At least 8

b. At least 10

c. At least 12

d. Majority of the regular members

8. An auditor's examination performed in accordance with generally accepted auditing standards generally

should

a. Be expected to provide assurance that illegal acts will be detected where internal control is effective.

b. Be relied upon to disclose violations of truth-in-lending act.

c. Encompass a plan to search actively for illegalities which relate to operating aspects.

d. Not be relied upon to provide assurance that illegal acts will be detected.

9. Which of the following statements is true?

a. It is usually equally difficult for the auditor to uncover errors or irregularities.

b. It is usually easier for the auditor to uncover irregularities than errors.

c. It is usually easier for the auditor to uncover errors than irregularit ies.

d. Usually, none of the above statements is true.

10. The audit should not assume that management is dishonest, but the possibility of dishonesty must be

considered." This is an example of

a. Due diligence.

b. Unprofessional behavior.

c. An attitude of professional skepticism.

d. An ethical requirement.

11. In discovering material management fraud and an equally material error, the audit plan

a. Cannot be expected to provide the same degree of assurance.

b. Provide no assurance of detecting either.

c. Should be expected to provide the same degree of assurance.

d. Should provide complete assurance of detection.

12. In comparing management fraud with employee fraud, the auditor's risk of failing to discover the fraud is

a. Greater for employee fraud because of the larger number of employees in the organization.

b. Greater for employee fraud because of the higher crime rate among blue collar workers.

c. Greater for management fraud because of management's ability to override exist ing

internal controls.

d. Greater for management fraud because managers are inherently smarter than employees.

13. The risk that the audit will fail to uncover a material misstatement is eliminated

a. When the auditor has complied with generally accepted auditing standards.

b. If client has good internal control.

c. If client follows generally accepted accounting principles.

d. Under no circumstances.

14. The practitioner’s report on an assurance engagement should always include the following, except

a. A description of the engagement and identification of the subject matter

Page 14: MCQs - prelims

Red Sirug Page 14

b. Identification of the standards under which the engagements was conducted.

c. Identification of the criteria.

d. Reference to the work of an expert.

15. Which of the following is required if the professional accountant uses experts who are not professional

accountants?

a. The professional accountant is discouraged to engage the services of experts who are not a

professional accountant.

b. The ultimate responsibility for the professional service is assumed by the expert who is not a

professional accountant.

c. The professional accountant must take steps to see that such experts are aware of the

ethical requirements of the profession.

d. Experts who are not professional accountants need not be informed of ethical requirements because

they are not members of the Accountancy profession.

16. Which of the following is expected of AASC to do?

a. AASC should normally expose its opinion on specific queries from a practicing CPA.

b. AASC should normally expose a proposed interpretation of statements.

c. To make the statements on Philippine Standards on Audit ing operative, the final

statement shall be submitted to the Board of Accountancy for approval.

d. When it is deemed necessary to expose a statement for a comment on proposed interpretations of

statements, the exposure period is understandably shorter than those of the regular drafts of

standards.

17. Required auditor communication to the Audit Committee concerning noncompliance with laws and

regulations that were detected includes:

a. All those which are not adequately addressed by management.

b. All those that constitute management fraud.

c. All material items.

d. Any of such acts.

18. The decision as to how much evidence to be accumulated for a given set of circumstances is:

a. Provided in the Philippines Standards on Auditing.

b. Provided by following GAAP

c. One requiring professional judgment

d. Determined by statistical analysis

19. The revised Code of Ethics is mandatory for all CPAs and is applicable to professional services performed in

the Philippines on or:

a. Before June 30. 2008

b. After June 30, 2008

c. Before January 1,2008

d. After January 1, 2008

20. Which of the following is not explicitly referred to in the Code of Ethics as source of technical standards?

a. Commission on Audit (COA)

b. Auditing and Assurance Standards Council (AASC)

c. Securities and Exchange Commission (SEC)

d. Relevant legislation

21. Firm includes the following except:

a. A sole practicing professional accountant,

b. An entity that controls a partnership of professional accountants.

c. An entity controlled by a partnership of professional accountants.

d. A sole practitioner, partnership or corporat ion of professional accountants.

22. The term professional accountant in public practice includes the following, except:

a. A sole proprietor providing professional services to a client.

b. Each partner or person occupying a position similar to that of a partner staff in a practice providing

professional services to a client.

c. Professional accountants employed in the public sector having managerial responsibilities.

d. A firm of professional accountants in public practice.

23. The term receiving accountant includes the following, except;

a. A professional accountant in public practice to whom existing accountant has referred tax engagement.

Page 15: MCQs - prelims

Red Sirug Page 15

b. A professional accountant in public practice to whom the client of the existing accountant has referred

audit engagement.

c. A professional accountant in public practice who is consulted in order to meet the needs of the client.

d. A professional accountant in public practice currently holding an audit appointment or

carrying out accounting, taxation, consulting or similar professional services for a client.

24. Related entity is an entity that has any of the following relationships with the client, except:

a. An entity that has direct or indirect control over the client provided that the client is material to such

entity.

b. An entity with a direct financial interest in the client even though such entity has no

significant influence over the client provided the interest in the client is material to such

entity.

c. An entity over which the client has direct or indirect control.

d. An entity which is under common control with that client (referred to as a "sister entity") provided the

sister entity and the client are both material to the entity that controls both the client and sister entity.

25. A primary purpose for establishing a code of ethics within a professional organization is to:

a. Demonstrate the acceptance of responsibility Jo the interest of those served by the

profession.

b. Reduce the likelihood that members of the profession will be sued for substandard work.

c. Ensure that all members of the profession posses approximately the same level of competency.

d. Require members of the profession to exhibit loyalty in all matters pertaining to the affairs of the

organization.

26. The communication to the public of facts about a professional accountant which are not designed for the

deliberate promotion of that professional accountant.

a. Publicity

b. Indirect promotion

c. Advertising

d. Solicitation

27. Advertising, as defined in the Code of Ethics, means

a. The communication to the public of facts about a professional accountant which are not designed for

the deliberate promotion of that professional accountant.

b. The approach to a potential client for the purpose of offering professional services.

c. The communicat ion to the public of information as to the services or skills provided by

professional accountants in public practice with a view to procuring professional business.

d. Any of the given choices.

28. Which of the following is least likely the basis of determining audit fees?

a. The skill and knowledge required for the type of work involved.

b. The degree of responsibility and urgency that the work entails.

c. The expected outcome of the engagement.

d. The required level of training and experience of the person engaged on the work.

29. Though PSAs do not provide "hard and fast rules," they provide subjective guidance which allow the

auditors to:

a. Only apply those standards that are important to the audit.

b. Use adequate professional judgment when applying the standards.

c. Tailor their audit to procedures requested by management.

d. Accurately interpret the Code of Ethics for CPAs.

30. Which one of the following is not a key attribute that is essential to perform an assurance service?

a. Independence

b. Established criteria or standards

c. Subject matter knowledge

d. Accounting skills

31. Upon completion of a typical audit, the auditor has

a. No assurance that all material errors and fraud have been found.

b. A low level of assurance that all material errors and fraud have been found.

c. High level of assurance that all material errors and fraud have been found.

d. Total assurance that al material errors and fraud have been found.

32. An investor, while reading the financial statements of Star Corporation, learned that the s tatements are

accompanied by an unqualified auditor's report. From this the investor may conclude that:

Page 16: MCQs - prelims

Red Sirug Page 16

a. The auditor has ascertained that Silver's financial statements have been prepared accurately.

b. The auditor is satisfied that Silver is operationally effi cient.

c. Any disputes over significant accounting issues have been settled to the auditor's

satisfaction.

d. Informative disclosures in the financial statements but not necessarily in the notes to financial

statements are to be regarded as reasonably adequate.

33. A CPA should maintain objectivity and be free of conflicts of interest when performing:

a. All attestation services, but not other professional services.

b. All attestation and tax services, but not other professional services.

c. Audits, but not any other professional services.

d. All professional services.

34. Which of the following has primary responsibility for the performance of an audit?

a. The partner in charge of the engagement

b. The senior assigned to the engagement

c. The managing partner of the firm

d. The manager assigned to the engagement

35. The most common type of audit report contains a(n):

a. Unqualified opinion.

b. Qualified opinion.

c. Adverse opinion.

d. Disclaimer of opinion.

36. In "auditing" accounting data, the auditor is concerned with

a. Analyzing the financial information to be sure that it complied with government requirement.

b. Determining if fraud has occurred.

c. Determining whether recorded information properly reflects the economic events that

occurred during the accounting period.

d. Determining if taxable income has been calculated correctly.

37. Which one of the following is an example of management expectations from the independent auditors?

a. An expert providing a written communicat ion as the product of the engagement.

b. Individuals who perform day-to-day accounting functions on behalf of the company.

c. AN active participant in management decision-making.

d. An internal source of expertise on financial and other matters.

38. Broadly defined, the subject matter of any audit consists of

a. Financial statements.

b. Assertions.

c. Operating data.

d. Economic data.

39. An engagement in which a CPA firm arranges for a critical review of its practices by another CPA firm.

a. Attestation engagement

b. Peer review engagement

c. Quality control engagement

d. Quality assurance engagement

40. The risk associated with a company's survival and profitability is referred to as:

a. Information risk

b. Business risk

c. Control risk

d. Detection risk

41. An operational audit differs in many ways from an audit of financial statements. Which of the following is

the best example of these differences?

a. Operation audits do not ordinarily result in the preparation of a report.

b. The usual audit of financial statement covers the four basic financial statements whereas the

operational audit is usually limited either the balance sheet or the income statement.

c. The boundaries of an operat ion audit are often drawn from an organizat ion chart and are

not limited to a single accounting period.

d. The operational audit deals with operating profit while financial audit considers both the operating and

net profits.

Page 17: MCQs - prelims

Red Sirug Page 17

42. The audit of historical financial statements should be conducted by the CPA professionals in accordance

with

a. The auditor's judgment.

b. The audit program.

c. Philippine Financial Reporting Standards.

d. Philippine Standards on Audit ing.

43. Whenever a CPA professional is engaged to perform an audit of financial statements according to

Philippine Standards on Auditing, he required to comply with those standards in order to

a. Eliminate audit risk.

b. Eliminate the professional judgment in resolving audit issues.

c. Have a measure of the quality of audit performance.

d. To reduce the audit program to be prepared by the auditor.

44. In determining the primary responsibility of the external auditor for an audit of a company's financial

statements, the auditor owes primary allegiance to:

a. The management of the audit client because the auditor is hired and paid by management.

b. The audit committee of the audit client because that committee is responsible for coordinating and

reviewing all audit activities within the company.

c. Stockholders, creditors, and the investing public.

d. The Auditing and Assurance Standards Council, because it determines auditing standards and auditor's

responsibility.

45. Assurance services involve all the following except:

a. Improving the quality of information for decision purposes.

b. Improving the quality of the decision model used.

c. Improving the relevance of information.

d. Implementing a system that improves the processing of information.

46. Which of the following is the broadest and most inclusive concept?

a. Audits of financial statements.

b. Internal control audit.

c. Assurance services.

d. Compilation services.

47. When performing an engagement to review a nonpublic entity's financial statements, an accountant most

likely would:

a. Obtain an understanding of the entity's internal control.

b. Limit the distribution of the accountant's report.

c. Confirm a sample of significant accounts receivable balances.

d. Ask about actions taken at board of directors' meetings.

48. Assurance services may include which of the following?

a. Attesting to financial statements

b. Examination of the economy and efficiency of governmental operations

c. Evaluation of a division's performance for management

d. All of the given choices

49. The auditor of financial statements must make very difficult interpretations regarding authoritative

literature. Additionally, the auditor must

a. Proceed beyond PFRS to assess how the economic activity is portrayed in the financial

statements.

b. Force management to make certain decisions regarding their financial statements.

c. Disregard independence in order to find the underlying truth of the evidence.

d. Establish new criteria by which financial statements may be compared.

50. Which one of the following is not a part of the attest process?

a. Evaluating evidence against objective criteria

b. Gathering evidence about assertions

c. Proving the accuracy of the books and records

d. Communicating the conclusions reached

51. Which one of the following is not a reason why the users of financial statements desire for an independent

assessment of the financial statement presentation?

a. Complexity of transactions affecting the financial statements

b. Lack of criteria on which to base information

Page 18: MCQs - prelims

Red Sirug Page 18

c. Remoteness of the user from the organization

d. All of them are potential reasons

52. An audit which determines whether organizational policies are being followed and whether external

mandates are being met is referred to as

a. A financial audit

b. A compliance audit.

c. An operational audit.

d. None of the above

53. Which of the following factors most likely would cause a CPA to decline a new audit engagement?

a. The CPA does not understand the entity's operations and industry.

b. The CPA is unable to review the predecessor auditor's working papers.

c. Management is unwilling to permit inquiry of its legal counsel.

d. Management acknowledges that the entity has had recurring operating losses.

54. Auditing standards are

a. Statutory in nature.

b. Rules imposed by the Securities and Exchange Commission.

c. General guidelines to help auditors.

d. Rules imposed by the PICPA.

55. Which of the following best describes what is meant by generally accepted auditing standards?

a. Pronouncements issued by the Auditing and Assurance Standards Council.

b. Rules acknowledged by the accounting profession because of their universal compliance.

c. Procedures to be used to gather evidence to support financial statements.

d. Measures of the quality of the auditor's performance.

56. Generally Accepted Auditing Standards (GAAS) and Philippine Standards on Auditing (PSA) should be

looked upon by practitioners as

a. Ideals to work towards, but which are not achievable.

b. Benchmarks to be used on all audits, reviews, and compilations.

c. Maximum standards which denote excellent work.

d. Minimum standards of performance which must be achieved on each audit engagement.

57. Competence as a certified public accountant includes all of the following except

a. Consulting others if additional technical information is needed.

b. Possessing the ability to supervise and evaluate the quality of staff work.

c. Having the technical qualifications to perform an engagement.

d. Warranting the infallibility of the work performed.

58. Which one of the following attributes is required of an auditor in relation to audit clients?

a. Loyalty

b. Rationalization

c. Independence

d. Bias

59. To be independent, the auditor:

a. Cannot place any reliance on the client's verbal and written assertions.

b. Is responsible only to third-party users of the financial statements.

c. Cannot perform any consulting services for an audit client.

d. Must be impart ial when dealing with the client.

60. Practitioner's independence:

a. Minimizes risk.

b. Defends against liability

c. Helps achieve public confidence

d. Achieves compliance with the standards of fieldwork.

61. The exercise of due professional care requires that an auditor

a. Uses error-free judgment.

b. Considers internal control, including tests of controls.

c. Examines all corroborating evidence available.

d. Be responsible for fulfilling his or her dut ies diligently and carefully.

62. The exercise of due professional care requires that an auditor

Page 19: MCQs - prelims

Red Sirug Page 19

a. Critically review the judgment exercised at every level of supervision.

b. Attain the proper balance of professional experience and formal education.

c. Reduce control risk below the maximum.

d. Examine all available corroborating evidence.

63. The exercise of due professional care requires that an auditor

a. Use error-free judgment.

b. Consider internal control, including tests of controls.

c. Critically review the work done at every level of supervision.

d. Examine all corroborating evidence available.

64. An auditor who accepts an audit engagement and does not possess the industry expertise of the business

entity, should

a. Engage financial experts familiar with the nature of the business entity.

b. Obtain a knowledge of matters that relates to the nature of the entity's business.

c. Refer a substantial portion of the audit to another CPA who will act as the principal auditor.

d. First inform management that an unqualified opinion cannot be issued.

65. Which of the following underlies the application of generally accepted auditing standards, particularly the

standards of fieldwork and reporting?

a. Element of corroborating evidence

b. Element of reasonable assurance

c. Elements of materiality and risk

d. Element of internal control

66. Which of the following is the authoritative body designated to promulgate auditing standards?

a. Financial Reporting Standards Council

b. PICPA

c. Association of CPAs in Public Practice and PICPA

d. Audit ing and Assurance Standards Council

67. Which of the following mostly describes the function of AASC?

a. To monitor full compliance by auditors to PSAs.

b. To assist the Board of Accountancy in conducting administrative proceedings on erring CPAs in audit

practice.

c. To promulgate audit ing standards, practices and procedures that shall be generally

accepted by the accounting profession in the Philippines.

d. To undertake continuing research on both auditing and financial accounting in order to make them

responsive to the needs of the public.

68. The Philippine Standards on Auditing issued by the Auditing and Assurance Standards Council (AASC)

a. Are interpretations of generally accepted auditing standards

b. Are the equivalent of laws for audit practitioners.

c. Must be followed in all situations.

d. Are optional guidelines which an auditor may choose to follow or not follow when conducting an audit.

69. Which of the following statements best describes the primary purpose of Philippines Standards on

Auditing?

a. They are authoritative statements, enforced through the Code of Ethics for Professional Accountants,

that are intended to limit the degree of auditor judgment.

b. They are procedural outlines which are intended to narrow the areas of inconsistency and divergence

of auditor opinion.

c. They are guides intended to set forth auditing procedures that are applicable to a variety of situations.

d. They are interpretations which are intended to clarify the meaning of "generally accepted

audit ing standards."

70. In financial statement audits, the audit process should be conducted in accordance with

a. Philippine Financial Reporting Standards

b. International Accounting Standards

c. Philippine Standards on Audit ing

d. Philippine Accounting Standards

71. The Philippine Standards on Auditing issued by AASC

a. Need to be applied on all audit related.

b. Require that in no circumstances would an auditor may judge it necessary to depart from a PSA, even

though such a departure may result to more effective achievement of the objective of an audit.

Page 20: MCQs - prelims

Red Sirug Page 20

c. Apply to independent examinat ion of financial statements of any ent ity when such an

examinat ion is conducted for the purpose of expressing an opinion.

d. Must not apply to other related activities of auditors.

72. An auditor needs not abide by a Philippines Standard on Auditing if the auditor believes that

a. The amount is insignificant.

b. The requirement of the PSA is impractical to perform.

c. The requirement of the PSA is impossible to perform.

d. Any of the above three is correct.

73. Auditing standards differ from auditing procedures in that procedures relate to:

a. Measures of performance

b. Acts to be performed.

c. Audit judgments.

d. Audit principles.

74. Every independent audit engagement involves both auditing standards and auditing procedures. The

relationship between the two may be illustrated by how they apply from engagement to engagement. The

best representation of this application is that, from one audit engagement to the next,

a. Audit ing standards are applied uniformly but audit ing procedures may vary.

b. Auditing standards may vary but auditing procedures are applied uniformly.

c. Auditing standards are applied uniformly but auditing procedures are optional.

d. Both auditing standards and auditing procedures are applied uniformly.

75. Generally accepted accounting principles (GAAP) are distinguished from generally accepted auditing

standards (GAAS) in that:

a. GAAP are the principles auditors follow when conducting an audit, while GAAS are the standards for

presentation of financial statements and underlying transactions.

b. GAAP are the principles for presentation of financial statements and underlying

transactions, while GAAS are the standards that the auditors should follow when

conducting an audit.

c. When GAAP are violated, sufficiently strong GAAS may make up for most GAAP deficiencies.

d. GAAP are promulgated by the SEC, while GAAS are promulgated by the PFRC.

76. Which of the following is not allowed by the revised code of ethics?

a. A professional accountant in public practice may issue to client or, in response to an unsolicited

request, to a non-client a factual and objectively worded of the services

provided.

b. Booklets and other documents bearing the name of a professional accountant and giving technical

information for the assistance of staff or clients may be issued to such persons, other professional

accountants or other interested parties.

c. The use of the name of an internat ional accounting firm affiliation/correspondence is

generally allowed.

d. A firm or CPA practitioner can continue to use the term "Accredited" or any similar words or phrase

calculated to convey the same meaning if the claimed accreditation has not expired.

77. How frequent can a professional accountants have press and other media releases commemorating their

anniversaries in public practice by informing the public of their achievements or accomplishments in

contributing toward nation building or enhancing the image or standards of the accounting profession?

a. 2 years

b. 3 years

c. 5 years

d. 6 years

78. Which of the following is not allowed to be included in a website of a firm of professional accountants?

a. Names of partners/principals with their educational attainment.

b. Membership in any professional body.

c. Awards received

d. Listings of the firm's clients.

79. In their fiduciary role, the professional accountants owe their primary loyalty to:

a. The accounting profession

b. The general public

c. The client

d. Government regulatory agencies

Page 21: MCQs - prelims

Red Sirug Page 21

80. Which of the following is a distinguishing mark of the accountancy profession?

a. A drive to excellence

b. Acceptance of the responsibility to act in the public interest

c. Professional objectivity

d. Professional skepticism

81. The IFAC Code of Professional Conduc t will ordinarily be considered to have been violated when the

member represents that specific consulting services will be performed for a stated fee and it is apparent at

the time of the representation that the

a. Actual fee would be substant ially higher.

b. Actual fee would be substantially lower than the fees charged by other members for comparable

services.

c. Fee was a competitive bid.

d. Member would not be independent.

82. A professional accountant is likened to a prudent father to his son. This relates to what fundamental

principle?

a. Professional competence and due care

b. Confidentiality

c. Integrity

d. Objectivity

83. Which fundamental principle is seriously threatened by an engagement that is compensated based on the

net proceeds on loans received by the client from a commercial bank?

a. Integrity

b. Objectivity

c. Professional behavior

d. Confidentiality

84. Which of the following values is not necessary for a professional accountant?

a. Honesty

b. Objectivity

c. Integrity

d. A primary commitment to self-interest

85. Professional accountants may encounter problems in identifying unethical behavior or in resolving an

ethical conflict. When faced with significant ethical issues, professional accountants should do the

following, except

a. Follow the established policies of the employing organization to seek a resolution of such conflict

b. Review the conflict problem with the immediate superior if the organization's policies do not resolve

the ethical conflict.

c. If the problem is not resolved with the immediate superior and the professional

accountant determines to go to the next higher managerial level, the immediate superior

need not be not ified of the decision.

d. Seek counseling and advice on a confidential basis with an independent advisor or the applicable

professional accountancy body or regulatory body to obtain an understanding of possible courses of

action.

86. As a resolution of the conflict in the application of fundamental principles, the auditor, after considering the

ethical issues and relevant facts may do any of the following, except:

a. Must immediately resign from the engagement or the employing entity.

b. Should weigh me consequences of each possible course of action.

c. Should consult with other appropriate persons within the firm or employing organization for help to

finally resolve the matter.

d. The professional accountant may wish to obtain professional advice from, the relevant professional

body without breaching confidentiality if significant conflict cannot be resolved.

87. Which of the following is incorrect regarding integrity and objectivity?

a. Integrity implies not merely honesty but fair dealing and truthfulness.

b. The principle of objectivity imposes the obligation on all professional accountants to be fair,

intellectually honest and free of conflicts of interest.

c. Professional accountants serve in many different capacities and should demonstrate their objectivity in

varying circumstances.

d. Professional accountants should neither accept nor offer any gifts or entertainment.

Page 22: MCQs - prelims

Red Sirug Page 22

88. If a professional accountant is billing an audit client a number of hours greater than those actually worked,

which of the following fundamental principles is likely violated?

a. Objectivity

b. Integrity

c. Professional due care

d. Confidentiality

89. Which of the following is not a function of the Board of Accountancy as specified in the Philippine

Accountancy Act of 2004?

a. To investigate violations of the Accountancy Law and the rules and regulations promulgated therewith.

b. To look from time to time into the conditions affecting the practice of the accountancy profession.

c. To create and direct accredit ing agencies that are entrusted the funct ions of reviewing

higher educat ional institut ions' policies and practices leading to

accreditation/reaccreditation of BSA program.

d. To determine and prescribe minimum requirements leading to the admission of candidates to the CPA

licensure examination.

90. All of the following are represented to the Financial Reporting Standards Council, except:

a. Securities and Exchange Commission

b. Bureau of Internal Revenue

c. Commission on Higher Education

d. Board of Accountancy

91. A CPA whose certificate of registration has been revoked:

a. Can no longer be reinstated.

b. Is automatically reinstated as a CPA by the PRC after two years if he has acted in an exemplary

manner.

c. May be reinstated by the Professional Regulation Commission after two years if he has acted in an

exemplary manner.

d. May be reinstated as a CPA by the Board of Accountancy after two years if he has acted in

an exemplary manner.

92. The Philippine Accountancy Act of 2004 provides that all Working papers made during an audit shall be the

property of the auditor. These working papers shall include the following, except:

a. Working papers prepared by the CPA and his staff.

b. Analysis and schedule prepared and submitted to the auditor by his client's staff.

c. Excerpts or copies of documents furnished to the auditor.

d. Any report submitted by the auditor to his client.

93. Who are required to apply for accreditation with the Professional Regulation Commission if the applicant is

a partnership of Professional Accountants?

a. Managing partner only

b. All partners only

c. Partners and staff members

d. Partners, principals, and staff members

94. Which of the following is not included in the seal of a professional accountant?

a. Tax identification number

b. Name of the professional accountant

c. Title of the profession

d. Registration number

95. Individual CPAs, Firms or Partnerships of CPAs, including partners and staff members thereof shall register

with the BOA and the PRC. If the accreditation of Velasco and Co., CPAs, was renewed on September 30,

2008, the next renewal must be on or before:

a. September 30, 2010

b. September 30, 2011

c. December 31, 2010

d. December 31, 2011

96. The APO shall renew its Certificate of Accreditation once every how many years after the date of the

Resolution granting the petition for re-accreditation and the issuance of the said certificate upon

submission of the requirements?

a. 2 years

b. 3 years

c. 4 years

Page 23: MCQs - prelims

Red Sirug Page 23

d. 6 years

97. Engagement letters are widely used in practice for professional engagements of all types. The primary

purpose of the engagement letter is to

a. Remind management of its primary responsibility over the financial statements.

b. Satisfy the requirements of the Code of Professional Conduct for CPAs.

c. Provide a starting point for the auditor's preparation of the preliminary audit program.

d. Provide a written record of the agreement with the client as to the services to be provided.

98. Which of the following is not likely a quality control procedure on consultation?

a. Identifies areas and specialized situations where consultation is required and encourages personnel to

consult with or in use authoritative sources on other complex matters.

b. Designates individuals as specialists to serve as authoritative sources and define their authority in

consultative situations.

c. Assigns an appropriate person or persons to be responsible for assigning personnel to

audits.

d. Specifies the extent of documentation to be provided for the result of consultation in those areas and

specialized situations where consultation is required.

99. According to Philippine Standards on Auditing, because there are inherent limitations in an audit that affect

the auditor's ability to detect material misstatements, the auditor is:

a. Neither a guarantor nor an insurer of financial statements.

b. A guarantor but not an insurer of the statements.

c. An insurer but not a guarantor of the statements.

d. Both a guarantor and an insurer of the financial statements.

100. The working papers prepared by a CPA in connection with an audit engagement are owned by the CPA,

subject to certain limitations. The rationale for this rule is to

a. Protect the working papers from being subpoenaed.

b. Provide the basis for excluding admission of the working papers as evidence because of the privileged

communication rule.

c. Provide the CPA with evidence and documentation which may be helpful in the event of a

lawsuit.

d. Establish a continuity of relationship with the client whereby indiscriminate replacement of CPAs is

discouraged.

SET 3

1. The responsibility for adopting sound accounting policies, maintaining adequate internal control, and

making fair representations in the financial statements rests

a. With the management.

b. With the independent auditor.

c. Equally with management and the auditor.

d. With the internal audit department.

2. The ordinary examination of financial statements is not primarily designed to disclose defalcations and

other irregularities although their discovery may result. Normal audit procedures are more likely to detect a

fraud arising from

a. Collusion on the part of several employees.

b. Failure to record cash receipts for services rendered.

c. Forgeries on company checks.

d. Theft of inventories.

3. A principal purpose of a letter of representation from management is to

a. Serve as an introduction to company personnel and an authorization to examine the records.

b. Discharge the auditor from legal liability for his examination.

c. Confirm in writing management's approval of limitations on the scope of the audit.

d. Remind management of its primary responsibility for financial statements.

4. The auditor should not assume that management is dishonest, but the possibility of dishonesty must be

considered." This is an example of

a. Expectation gap.

b. An attitude of professional skepticism.

c. Due diligence.

d. An ethical requirement.

Page 24: MCQs - prelims

Red Sirug Page 24

5. Which of the following statements is true?

a. It is usually easier for the auditor to uncover irregularities than errors.

b. It is usually easier for the auditor to uncover errors than irregularit ies.

c. It is usually equally difficult for the auditor to uncover errors or irregularities.

d. Usually, none of the given statements is true.

6. Generally, the decision to notify parties outside the client's organization regarding a noncompliance with

laws and regulations

a. Independent auditor.

b. Management.

c. Outside legal counsel.

d. Internal auditors.

7. An audit made in accordance with Philippine Standards on Auditing generally should

a. Be expected to provide assurance that noncompliance with laws and regulations will be detected if the

internal control is effective.

b. Be relied upon to disclose indirect-effect noncompliance with laws and regulations.

c. Encompass a plan to search actively for noncompliance with laws and regulations which relate to

operating aspects.

d. Not be relied upon to provide assurance that all noncompliance with laws and regulations

will be detected.

8. An auditor who believes that a material irregularity may exist should initially

a. Discuss the matter with those believed to be involved in the perpetration of material irregularity.

b. Discuss the matter with a higher level of management.

c. Withdraw from the engagement.

d. Consult legal counsel.

9. When management refuses to disclose in the financial statements noncompliance to laws and regulations

which are identified by the independent auditor, the CPA may be charged with unethical conduct for

a. Withdrawing from the engagement.

b. Issuing a disclaimer of opinion.

c. Failure to uncover the noncompliance to laws and regulations during the prior audits.

d. Reporting these activities to the audit committee.

10. A procedure in which a quality control partner periodically tests the application of quality control

procedures is most directly related to which quality control element?

a. Engagement performance

b. Independence, integrity, and objectivity

c. Monitoring

d. Personnel management

11. The work of each assistant needs to be reviewed by personnel of at least equal competence. Which of the

following is not one of the objectives of this requirement?

a. The conclusions expressed are consistent with the result of the work performed and support the

opinion.

b. The work performed and the results obtained have been adequately documented.

c. The audit objectives have been achieved.

d. All available evidences have been obtained, evaluated and documented.

12. Which of the following is incorrect regarding the professional accountants' tax practice?

a. A professional accountant rendering professional tax services is entitled to put forward the best

position in favor of a client, or an employer.

b. Doubt may be resolved in favor of the client or the employer if there is a reasonable support for the

position.

c. A professional accountant may hold out to a client or an employer the assurance that the

tax return prepared and the tax advice offered by him are beyond challenge.

d. Professional accountants should ensure that the client or the employer is aware of the limitations

attaching to tax advice and services so that they do not misinterpret an expression of opinion as an

assertion of fact.

13. Which of the following is least likely an application of maintaining an attitude of professional skepticism?

a. The auditor does not consider representations from management as substitute for obtaining sufficient

appropriate audit evidence to be able to draw reasonable conclusions on which to base the audit

opinion.

b. In planning and performing an audit, the auditor assumes that management is dishonest.

Page 25: MCQs - prelims

Red Sirug Page 25

c. The auditor is alert to audit evidence that contradicts or brings into question the reliability of

documents or management representations.

d. The auditor makes a critical assessment, with a questioning mind, of the validity of audit evidence

obtained.

14. Which of the following statements is true when the CPA has been engaged to do an attestation

engagement?

a. The CPA firm is engaged and paid by the client; therefore, the firm has primary responsibility to be an

advocate for the client.

b. The CPA firm is engaged and paid by the client, but the primary beneficiaries of the audit

are the statement users.

c. Should a situation arise where there is no convincing authoritative standard available, and there is a

choice of actions which could impact client's financial statements either positively or negatively, the

CPA is free to endorse the choice which is best in the client's interest.

d. As long as CPA firms are competent, it is not required that they remain unbiased.

15. Which of the following will impair the independence of a CPA in public practice?

a. He has his name and address listed on a one-page section of the telephone book.

b. He obtained a loan from a bank under the normal lending procedures, terms and requirements of the

bank.

c. He holds one share of the client's capital stock.

d. He failed to disclose a client's departure from GAAP.

16. When CPAs are able to maintain an independent attitude in fulfilling their responsibility, it is referred to as

independence in

a. Fact.

b. Appearance.

c. Conduct.

d. Total.

17. When the users of financial statements have confidence in the independence of the CPA, it is referred to as

in independence in

a. Fact.

b. Appearance.

c. Conduct.

d. Total.

18. When determining whether independence is impaired because of an ownership interest in client company,

materiality will affect whether ownership is a violation of rule of independence

a. In all circumstances.

b. Only for direct ownership.

c. Only for indirect ownership.

d. Under no circumstances.

19. A professional accountant has a professional duty or right disclose confidential information in each of the

following, except:

a. To comply with technical standards and ethics requirements.

b. To disclose to the Bureau of Internal Revenue any fraudulent scheme committed by the

client on payment of income tax.

c. To comply with the quality review of a member body or professional body

d. To respond to an inquiry or investigation by a member body or regulatory body.

20. Which of the following best describes the passing of confidential information from a client to its auditor?

The information:

a. Should in no circumstances be conveyed to third parties.

b. Is not legally protected and can be subpoenaed by a competent court.

c. Can only be released for peer reviews after receiving permission from the client.

d. Should be conveyed to the public if it affects the "correctness" of the financial statements.

21. A member in public practice may perform for a contingent fee any professional services for a client for

whom the member or member's firm performs

a. An audit.

b. A review.

c. A compilat ion used only by management.

d. An audit of prospective financial information.

Page 26: MCQs - prelims

Red Sirug Page 26

22. Which one of the following contingent fee is allowed?

a. All services performed by a CPA film.

b. Non-attestation services.

c. Non-attestation services, unless the CPA firm was also performing attestation services for

the same client.

d. Attestation services.

23. When the auditor issues an erroneous opinion as a consequence of an underlying failure to comply with

the requirements of generally accepted auditing standards, it results to

a. Business failure.

b. Audit failure.

c. Audit risk.

d. All of them

24. The responsibility for the fairness of the financial assertions that are embodied in the financial statements

and in the notes to the financial statements rests:

a. With the audit committee.

b. With management.

c. With Securities and Exchange Commission.

d. Equally with management and the stockholders.

25. When preparing the financial statements, it is acceptable for the auditor to prepare

a. The footnotes for client.

b. The statement for client.

c. A draft of the statements and footnotes for client.

d. A draft of the statements for client.

26. Which of the following statements best describes the auditor's responsibility regarding the detection of

material errors and frauds?

a. The auditor is responsible for the failure to detect material errors and frauds only when such failure

results from the misapplication of generally accepted accounting principles.

b. The auditor is responsible for the failure to detect material errors and frauds only when the auditor

fails to confirm receivables or observe inventories.

c. The audit should be designed to provide reasonable assurance that material errors and

frauds are detected.

d. Extended auditing procedures are required to detect unrecorded transactions even if there is no

evidence that material errors and frauds may exist.

27. The auditor has considerable responsibil ity for notifying users as to whether or not the statements are

properly stated. This imposes upon the auditor a duty to

a. Provide reasonable assurance that material misstatements will be detected.

b. Be an insurer of the fairness in the statements.

c. Be a guarantor of the fairness in the statements.

d. Be equally responsible with management for the preparation of the financial statements.

28. The factor that distinguishes an error from an irregularity is

a. Whether it is peso amount or a process.

b. Whether it is a caused by the auditor or the client.

c. Materiality.

d. Intent.

29. The reason why an auditor accumulates evidence is to

a. Defend himself in the event of a lawsuit.

b. Justify the conclusions he has otherwise reached.

c. Satisfy the requirements of the bureau of internal revenue.

d. Enable him to reach conclusions about the fairness of the financial statements and issue

an appropriate audit report.

30. The auditor gives an audit opinion on the fair presentation of the financial statements and associates his or

her name with them when, on the basis of adequate evidence, the auditor concludes that the financial

statements are unlikely to mislead

a. A prudent user.

b. Management.

c. The reader.

d. Investors.

Page 27: MCQs - prelims

Red Sirug Page 27

31. The type of transactions that ordinarily have a high inherent because they involve management judgment

or assumptions referred to as

a. Est imation transactions.

b. Nonroutine transactions.

c. Routine transactions.

d. Related-party transactions.

32. The probability that an auditor's procedures leading to the conclusion that a material error does not exist in

an account balance when, in fact, such error does exist is referred to as

a. Prevention risk.

b. Inherent risk.

c. Control risk.

d. Detection risk.

33. Which of the following is not included in an audit engagement letter?

a. Objectives of the engagement

b. Representations that the financial statements were prepared in accordance with PFRS

c. Management's responsibil ities

d. A clear explanation of the services to be performed on the engagement

34. Which of the following is least likely included in an auditor’s inquiry of management while obtaining

information to identify the risks of material misstatement due to fraud?

a. Are financial report ing operations controlled by and limited to one location?

b. Does it have knowledge of fraud or suspect fraud?

c. Does it have programs to mitigate fraud risks?

d. Has it reported to the audit committee the nature of the company's internal control?

35. Which of the following should the auditors normally interview as part of their assessment of fraud risk?

a. Senior management

b. Audit committee

c. Various employees whose duties financial reporting responsibil ities

d. All of the given choices

36. An audit plan is a

a. Detailed plan of analytical procedures and all substantive tests to be performed in the course of the

audit.

b. Document that provides an overview of the company and a general plan for the audit work

to be accomplished, t iming of the work, and other matters of concern to the audit.

c. Generic document that auditing firms have developed to lead the process of the audit through a

systematic and logical process.

d. Budget of the time that should be necessary to complete each phase of the audit procedures.

37. What will an auditor who has been proposed for an audit engagement usually do prior to accepting a new

client?

a. Draft the financial statements of the client as a measure of goodwill.

b. With the permission from the prospect ive client, contact the predecessor auditor to

determine if there are any disagreements between the client and the audit firms.

c. Obtain the potential client's permission to talk to the former auditor and review work papers.

d. Perform a peer review on the potential client in accordance with professional standards.

38. If the auditor sets the preliminary judgment about materiality level at a relatively low peso amount,

a. More evidence will be required than for a high level.

b. Less evidence will be required than for a high level.

c. The same amount of evidence will be required as for a high level.

d. The amount of evidence required will not be affected.

39. Which of the following statements is true with regard to the relationship among audit risk, audit evidence,

and materiality?

a. The lower the inherent risk and control risk, the lower the aggregate materiality threshold.

b. Under conditions of high inherent and control risk, the auditor should place more

emphasis on obtaining external evidence and should reduce reliance on internal evidence.

c. Where inherent risk is high and control risk is low, the auditor may safel y ignore inherent risk.

d. Aggregate materiality thresholds should not change under conditions of changing risk levels.

40. Which of the following is most likely to be an overall response to fraud risks identified in an audit?

a. Supervise members of the audit team less closely and rely more upon judgment.

Page 28: MCQs - prelims

Red Sirug Page 28

b. Use less predictable audit procedures.

c. Use only certified public accountants on the engagement.

d. Place increased emphasis on the audit of objective transactions rather than subjective transactions.

41. Which of the following represents a procedure that the auditor may use because plausible relationships

among financial statement balances are expected to exist?

a. Attributes testing

b. Enterprise risk assessment

c. Inherent tests of control

d. Analytical review

42. Which of the following statements is incorrect regarding obtaining an understanding of the entity and its

environment?

a. Obtaining an understanding of the entity and its environment is an essential aspect of performing an

audit in accordance with PSAs.

b. Understanding of the entity and its environment establishes frame of reference within which the

auditor plans the audit and exercises professional judgment about assessing risks of material

misstatement in the financial statements and responding to those risks throughout the audit.

c. The auditor's primary consideration is whether the understanding that has been obtained is sufficient

to assess the risks of material misstatement in the financial statements and to design and perform

further audit procedures.

d. The depth of the overall understanding that is required by the auditor in performing the

audit is at least equal to that possessed by management in managing the entity.

43. Which statement is incorrect regarding analytical procedures?

a. Analytical procedures may be helpful in identifying the existence of unusual transactions or events, and

amounts, ratios, and trends that might indicate matters that have financial statement and audit

implications.

b. In performing analytical procedures as risk assessment procedures, the auditor develops expectations

about plausible relationships that are reasonably expected to exist.

c. When comparison of those expectations with recorded amounts or ratios developed from recorded

amounts yields unusual or unexpected relationships, the auditor considers those results in identifying

risks of material misstatement.

d. When such analytical procedures use data aggregated at a high level (which is often the

situat ion), the results of those analytical procedures provide a clear-cut indicat ion whether a material misstatement may exist.

44. Inquiries directed towards those charged with governance may most likely

a. Relate to their activities concerning the design, and effectiveness of the entity's internal control and

whether management has satisfactorily responded to any findi ngs from these activities.

b. Help the auditor understand the environment in which the financial statements are

prepared.

c. Relate to changes in the entity's marketing strategies, sales trends, or contractual arrangements with

its customers.

d. Help the auditor in evaluating the appropriateness of the selection and application of certain

accounting policies.

45. The underlying reason for a code of professional conduct for any profession is

a. That it is required by congress.

b. The need for public confidence in the quality of service of the profession.

c. That it provides a safeguard to keep unscrupulous people out.

d. That it allows Professional Regulation Commission to have a yardstick to measure deficient

performance.

46. Professional accountants may encounter problems in identifying unethical behavior or in resolving an

ethical conflict. When faced with significant ethical issues, professional accountants should do the

following, except

a. Follow the established policies of the employing organization to seek a resolution of such conflict.

b. Review the conflict problem with the immediate superior if the organization's policies do not resolve

the ethical conflict.

c. If the problem is not resolved with the immediate superior and the professional

accountant determines to go to the next higher managerial level, the immediate superior

need not be not ified of the decision.

d. Seek counseling and advice on a confidential basis with an independent advisor or the applicable

professional accountancy body or regulatory body to obtain an understanding of possible courses of

action.

Page 29: MCQs - prelims

Red Sirug Page 29

47. The assessment of the risks of material misstatement at financial statement level is affected by the

auditor’s understanding of the control environment. Weaknesses control environment ordinarily will lead

the auditor to

a. Have more confidence in internal control and the reliability audit evidence generated internally within

the entity.

b. Conduct some audit procedures at an interim date rather than at period end.

c. Modify the nature of audit procedures to obtain more persuasive audit evidence.

d. Decrease the number of locations to be included in the audit scope.

48. The auditor should determine overall responses to address the risks of material misstatement at the

financial statement level. Such responses least likely include

a. Emphasizing to the audit team the need to maintain professional skepticism in gathering and

evaluating audit evidence.

b. Assigning more experienced staff or those with special skills or using experts.

c. Incorporating additional elements of unpredictability in the selection of further audit procedures to be

performed.

d. Performing substant ive procedures at an interim date instead of at period end.

49. The auditor should design and perform further audit procedures whose nature, timing, and extent are

responsive to the assessed risks of material misstatement at the assertion level. Which of the following is

the most important consideration in responding to the assessed risks?

a. The nature of the audit procedures.

b. The timing of the audit procedures.

c. The extent of the audit procedures.

d. All of these are equally important.

50. While assessing the risk of material misstatement, the auditors identity risks, relate risk to what could go

wrong, consider the magnitude of risks and:

a. Assess the risk of misstatements due to noncompliance to laws and regulations.

b. Consider the complexity of the transactions involved.

c. Consider the likelihood that the risks could result in material misstatements.

d. Determine materiality level.

51. Which of the following is least likely considered a financial statement audit risk factor?

a. Management operating and financing decisions are dominated by top management.

b. A new client with no prior audit history.

c. Rate of change in the entity's industry is rapid.

d. Profitability of the entity relative to its industry is inconsistent.

52. Which of the following is most likely to be considered a risk factor relating to fraudulent financial reporting?

a. Low turnover of senior management.

b. Extreme degree of competit ion within the industry.

c. Capital structure including various operating subsidiaries.

d. Sales goals in excess of any of the preceding three years.

53. Which of the following is correct concerning requirements about auditor's communications about fraud?

a. Fraud that involves senior management should be reported direct ly to the audit committee

regardless of the amounts involved.

b. All fraud with a material effect on the financial statements should be reported directly by the auditor to

the SEC

c. Fraud with a material effect on the financial statements should ordinarily be disclosed by the auditor

through the use of an emphasis of a matter paragraph added to the audit report.

d. The auditor has no responsibility to disclose fraud outside the entity under any circumstances.

54. Which of the following factors most likely would heighten an auditor's concern about the risk of fraudulent

financial reporting?

a. Large amounts of liquid assets that are easily convertible into cash.

b. Low growth and profitability as compared to other entity's in the same industry.

c. Financial management's participation in the initial selection of accounting principles.

d. An overly complex organizat ional structure involving unusual lines of authority.

55. Which of the following is most likely to be an overall response to fraud risks identified in an audit?

a. Only use certified public accountants on the engagement.

b. Place increased emphasis on the audit of objective transactions rather than subjective transactions.

c. Supervise members of the audit team less closely and rely more upon judgment.

Page 30: MCQs - prelims

Red Sirug Page 30

d. Use less predictable audit procedures.

56. There is generally an agreement within the auditing profession and the courts that the auditor

a. Is both a guarantor and an insurer of the financial statements.

b. Is a guarantor but not an insurer of the statements.

c. Is an insurer but not a guarantor of the statements.

d. Is neither a guarantor nor an insurer of financial statements.

57. Which of the following is correct?

a. The evidence which the auditor accumulates remains the same from audit to audit, but the general

objectives vary, depending on the circumstances.

b. The general audit objectives remain the same from audit to audit, but the evidence varies,

depending on the circumstances.

c. The circumstances may vary form audit to audit, but the evidence accumulated remains the same.

d. The general audit objectives may vary from audit to audit, but the circumstances remain the same.

58. The auditor is not liable to his client for

a. Negligence.

b. Bad faith.

c. Errors of judgment

d. Dishonesty.

59. A basic objective of a CPA firm is to provide professional services that conform to professional standards.

Reasonable assurance of achieving this basic objective is provided through

a. A system of peer review.

b. Continuing professional education.

c. Compliance with generally accepted reporting standards.

d. A system of quality control.

60. Which of the following statements regarding quali ty control policies and procedures is incorrect?

a. Quality control policies and procedures should be implemented at both the level of the audit firm and

on an individual audits.

b. The audit firm should implement quality control policies and procedures designed to ensure that all

audits are conducted in accordance with PSAs or relevant national standards or practices.

c. Quality control policies are objectives and goals while quality control procedures are steps to be taken

to accomplish the policies adopted.

d. The policies and procedures adopted by individual audit firms should not vary since there

is an applicable PSA that prescribes quality control policies and procedures that must be

adopted by all auditing firms.

61. A principal purpose of a letter of representation from management is to

a. Remind management of its primary responsibility for financial statements.

b. Serve as an introduction to company personnel and an authorization to examine the records.

c. Discharge the auditor from legal liability for his examination.

d. Confirm in writing management's approval of limitations on the scope of the audit.

62. Should the auditor uncover circumstances during the audit that may cause suspicions of management

fraud, the auditor must

a. Withdraw from engagement.

b. Issue an adverse opinion.

c. Issue a disclaimer.

d. Evaluate their implications and consider the need to modify audit evidence.

63. Most accounting and auditing professionals agree that when an audit has failed to uncover material

misstatements, and the wrong type of audit opinion is issued, the audit firm

a. Has failed to follow generally accepted auditing standards (GAAS).

b. Should be asked to defend the quality of the audit.

c. Deserves to lose the lawsuit.

d. Should not be held responsible for the financial loss suffered loss suffered by others.

64. A CPA firm studies its personnel advancement experience to ascertain whether individuals meeting stated

criteria are assigned increased degrees of responsibility. This is evidence of the firm's adherence to

prescribed standards of

a. Supervision and review.

b. Continuing professional education.

c. Professional development.

Page 31: MCQs - prelims

Red Sirug Page 31

d. Quality control.

65. Which of the following is not likely a quality control procedure on consultation?

a. Identifies areas and specialized situations where consultation is required and encourages personnel to

consult with or in use authoritative sources on other complex matters.

b. Designates individuals as specialists to serve as authoritative sources and define their authority in

consultative situations.

c. Specifies the extent of documentation to be provided for the result of consultation in those areas and

specialized situations where consultation is required.

d. Assigns an appropriate person or persons to be responsible for assigning personnel to

audits.

66. The objective of quality control mandates that a public accounting firm should establish policies and

procedures for professional development which provide reasonable assurance that all entry-level personnel

a. Prepare working papers which are standardized in form and content.

b. Will advance within the organization.

c. Develop specialties in specific areas of public accounting.

d. Have the knowledge required to enable them to fulfill responsibilit ies assigned.

67. Which of the following is an element of "directing an audit assistant" objective?

a. Identifying in advance the staffing requirements of a particular audit engagement.

b. Resolving any differences in professional judgment between audit personnel.

c. Resolution of differences in audit findings.

d. Informing assistants of their responsibilities and the objectives of the procedures they

are to perform.

68. In pursuing its quality control objectives with respect to assigning personnel to engagements, a public

accounting firm may use policies and procedures such as

a. Rotating employees from assignment to assignment on a random basis to aid in the staff training

effort.

b. Allowing staff to select the assignments of their choice to promote better client relationships.

c. Assigning a number of employees to each engagement in excess of the number required so as not to

overburden the staff, and interfere with the quality of the audit work performed.

d. Requiring t imely ident ificat ion of the staffing requirements of specific engagements so

that enough qualified personnel can be made available.

69. In connection with the element of professional development, a CPA firm's system of quality control should

ordinarily provide that all personnel

a. Possess judgment, motivation, and adequate experience.

b. Seek assistance from persons having appropriate level of knowledge, judgment, and authority.

c. Demonstrate compliance with peer review directives.

d. Have the knowledge required to enable them to fulfill responsibilit ies assigned.

70. A CPA establishes quality control policies and procedures for deciding whether to accept a new client or

continue to perform services for a current client. The primary purpose for establishing such policies and

procedures is

a. To enable the auditor to attest to the integrity or reliability of a client.

b. To comply with the quality control standards established by regulatory bodies.

c. To lessen the exposure to litigation resulting from failure to detect irregularities in client financial

statements.

d. To minimize the likelihood of association with clients whose management lacks integrity.

71. The implementation of quality control procedures that are applicable to the individual audit engagement is

the responsibility of the

a. CPA firm.

b. Engagement quality control reviewer.

c. Expert contracted by the firm in connection with the audit engagement.

d. Engagement team.

72. It involves a study or evaluation of the quality of audit of financial statements through a review of quality

control measures established by on CPA firms and individual CPAs in public practice to ensure compliance

with accounting and auditing standards and practices

a. External audit

b. Compliance audit

c. Peer review

d. Quality review

Page 32: MCQs - prelims

Red Sirug Page 32

73. Which of the following risks is entirely a quality criterion based on professional judgment?

a. Inherent risk

b. Control risk

c. Detection risk

d. Audit risk

74. The concept of materiality is not used by auditors as a guide to

a. Making decisions about the audit report

b. Planning the audit program

c. Evaluation of the audit evidence

d. Applicat ion of general standards

75. Inherent risk is not a characteristic of the

a. Major types of transactions.

b. Client’s business.

c. Substantive procedures.

d. Effectiveness of the client’s accountants.

76. Misstatements must be compared to some measurement base before a decision can be made about the

materiality of the failure to follow GAAP. A commonly accepted measurement base would be

a. Net income.

b. Total assets.

c. Working capital.

d. All of the above.

77. The primary deliverable of an engagement to perform based on procedures prescribed by the intended

user of the report is/are:

a. The Review Report

b. Report of Factual Findings

c. Management Letter

d. The financial statements

78. Fraudulent financial reporting is often called:

a. Theft of assets

b. Employee fraud

c. Management fraud

d. Defalcation

79. The ordinary examination of financial statements is not primarily designed to disclose defalcations and

other irregularities although their discovery may result. Normal audit procedures are more likely to detect

a fraud arising from

a. Theft of inventories.

b. Collusion on the part of several employees.

c. Failure to record cash receipts for services rendered.

d. Forgeries on company checks.

80. Which of the following is one of the functions of QCR:

a. To adopt a Code of Ethics for the practice of accountancy.

b. To promulgate accounting and auditing standards that will be generally accepted in the Philippines.

c. To evaluate periodically the performance of educational institutions offering accountancy education.

d. To conduct a review on applicants for registration to practice public accountancy and

render a report which shall be attached to the application for registration.

81. Working papers prepared by a CPA in connection with an audit engagement are owned by t he CPA, subject

to certain limitations. The rationale for this rule is to

a. Protect the working papers from, being subpoenaed.

b. Provide the CPA with evidence and documentation which may be helpful in the event of a

lawsuit.

c. Provide the basis for excluding admission of the working papers as evidence because of the privileged

communication rule.

d. Establish a continuity of relationship with the client whereby indiscriminate replacement of CPAs is

discouraged.

82. The auditor's responsibility for failure to detect fraud arises

a. Whenever the amounts involved are material.

Page 33: MCQs - prelims

Red Sirug Page 33

b. When such failure clearly results from noncompliance to generally accepted audit ing

standards.

c. Only when such failure clearly results from negligence so gross as to sustain an inference of fraud on

the part of the auditor.

d. Only when the examination was specifically designed to detect fraud.

83. The factor that distinguishes constructive fraud from actual fraud is

a. Materiality.

b. Quality of internal control.

c. Type of error or irregularity.

d. Intent.

84. If a CPA recklessly abandons standards of due care and diligence while performing an audit, he or she may

be held liable to unknown third parties for:

a. Gross negligence.

b. Fraudulent misconduct.

c. Gross misconduct.

d. Contributory negligence.

85. Of the following statements, which best distinguishes ordinary negligence from gross negligence?

a. The more material the undetected error the greater the likelihood of ordinary negligence.

b. Gross negligence is most probable when the auditor fails to detect errors that occurred under

conditions of strong internal control.

c. Failure to detect material errors, whether internal control is strong or weak, suggests gross negligence.

d. Failure to exercise reasonable care denotes ordinary negligence, whereas failure to

exercise minimal care indicates gross negligence.

86. The auditor is most likely to presume that a high risk of irregularities exists if

a. The client does business with several related parties.

b. The client is a multinational company that does business in numerous foreign countries.

c. Inadequate segregation of dut ies places an employee in a position to perpetrate and

conceal thefts.

d. Inadequate employee training results in lengthy EDP exception reports each month.

87. An auditor who believes that a material irregularity may exist should initially

a. Consult legal counsel.

b. Discuss the matter with those believed to be involved in the

perpetration of the material irregularity.

c. Discuss the matter with a higher level of management.

d. Withdraw from the engagement.

88. Which of the following statements is correct concerning the auditor's responsibility with respect to illegal

acts?

a. An auditor must design tests to detect both direct-effect and indirect-effect illegal acts.

b. An auditor must design tests to detect both immaterial and material direct-effect illegal acts.

c. An auditor must design tests to obtain reasonable assurance of detecting material direct -

effect illegal acts.

d. An auditor must design tests to detect both material direct-effect and material indirect-effect illegal

acts.

89. If specific information comes to an auditor's attention that implies the existence of possible illegal acts that

could have a material, but indirect effect on-the financial statements, the auditor should next

a. Report the matter to an appropriate level of management at least one level above those involved.

b. Apply audit procedures specifically directed to ascertaining whether an illegal act has

occurred.

c. Seek the advice of an informed expert qualified to practice law as to possible contingent liabilities.

d. Discuss the evidence with the client's audit committee, or others with equivalent authority.

90. When planning the audit, if the auditor has no reason to believe that illegal acts exist, the auditor should

a. Include audit procedures which have a strong probability of detecting illegal acts.

b. Make inquiries of management regarding their policies and regarding their knowledge of

violat ions, and then rely on normal audit procedures to detect errors, irregularities, and

illegalit ies.

c. Still include some audit procedures designed specifically to uncover illegalities.

d. Ignore the topic.

Page 34: MCQs - prelims

Red Sirug Page 34

91. When management refuses to disclose illegal activities which were identified by the independent auditor,

the independent auditor may be charged with unethical conduct for

a. Issuing a disclaimer of opinion.

b. Withdrawing from the engagement.

c. Failure to uncover the il legal activities during prior audits.

d. Reporting these activities to the audit committee.

92. What is the independent auditor's responsibility prior to completion of fieldwork when he believes that a

material fraud may have occurred?

a. Notify the appropriate law enforcement authority.

b. Continue to perform normal audit procedures and write the audit report in such a way to disclose

adequately the suspicions of material fraud.

c. Investigate the persons involved, the nature of the fraud, and the amounts involved.

d. Reach an understanding with the appropriate client representatives as to the desired

nature and extent of subsequent audit work.

93. An auditor who finds that the client has committed an illegal act would be most likely to withdraw from the

engagement when the

a. Illegal act has received widespread publicity.

b. Illegal act affects the auditor's ability to rely on management representations.

c. Illegal act has material financial statement implications.

d. Auditor cannot reasonably estimate the effect of the illegal act on the financial statements.

94. When the auditor knows that an illegal act has occurred, the auditor must

a. Issue an adverse opinion.

b. Withdraw from the engagement.

c. Report it to the proper government authorities.

d. Consider the effects on the financial statements, including the adequacy of disclosure.

95. When an independent auditor's examination of financial statements discloses special circumstances that

make the auditor suspect that fraud may exist, the auditor's initial course of action should be to

a. Decide whether the fraud, if in fact it should exist, might be of such a magnitude as to

affect the auditor's report on the financial statements.

b. Recommend that the client pursue the suspected fraud to a conclusion that is agreeable to the auditor.

c. Reach an understanding with the proper client representative as to whether the auditor or the client is

to make the investigation necessary to determine if a fraud has in fact occurred.

d. Extend normal audit procedures in an attempt to detect the full extent of the suspected fraud.

96. The auditor's evaluation of the likelihood of material employee fraud is normally done initially as a part of

a. The assessment of whether to accept the audit engagement.

b. Understanding the ent ity's internal control structure.

c. The Tests of Controls.

d. The Tests of Transactions.

97. In which circumstance is a CPA firm's independence most likely to be impaired?

a. An individual on the audit has a close relative who is a receptionist for the client.

b. The father of the audit senior holds a material financial interest in the client of which the senior is

unaware.

c. The spouse of a staff member on the audit has an immaterial common stock investment in

the audit client.

d. The partner in charge of the office's compensation is affected by office profi tability, a portion of which

arises from this audit.

98. Which of the following partners is least likely to be considered a "covered member" for purposes of

rendering assurance service to of Company A, a nonaudit client, performed by the head office of a national

CPA firm?

a. The partner in charge of the entire CPA firm.

b. A partner in the Cebu office of the CPA firm who maintains a small, immaterial investment

in Company A.

c. A partner in the Davao office who worked on the Company A for a different assurance engagement in

previous years, but currently has no responsibil ities with respect to the engagement

d. The partner in charge of the Davao office.

99. In a common law action against an accountant, the lack of privity is a viable defense if the plaintiff

a. Bases his action upon fraud.

b. Is the accountant's client.

Page 35: MCQs - prelims

Red Sirug Page 35

c. Is a creditor of the client who sues the accountant for negligence.

d. Can prove the presence of gross negligence which amounts to a reckless disregard for the truth.

100. A third party sues a public accounting firm for negligence under common law on the basis of materially

false financial statements. Which of the following is the firm’s defense?

a. Lack of reliance

b. Lack of privity

c. Lack of intent

d. Contributory negligence

101. The factor that distinguishes constructive fraud from ordinary negligence is

a. Type of error or irregularity.

b. Intent.

c. Level of care.

d. Materiality.

102. While performing services for their clients, professionals have always had a duty to provide a level of care

which is

a. Reasonable.

b. Greater than average.

c. Superior.

d. Guaranteed to be free from error.

103. The existence of extreme or unusual negligence, even though there was no intent to deceive or do harm,

is

a. Fraud.

b. Gross fraud.

c. Constructive fraud.

d. Ordinary fraud.

104. The failure of the auditor to meet generally accepted auditing standards is

a. An accepted practice.

b. A suggestion of negligence.

c. An evidence of negligence.

d. Tantamount to criminal behavior.

105. Which of the following statement/s is true?

a. Gross negligence may constitute constructive fraud.

b. Constructive fraud is also termed recklessness.

c. Fraud requires the intent to deceive.

d. All three above are true.

106. Which of the following, if present, would support a finding of

constructive fraud on the part of a CPA?

a. Privity of contract

b. Intent to deceive

c. Reckless disregard

d. Ordinary negligence

107. In rare cases auditors have been, held liable for criminal acts. A criminal conviction against an auditor can

result only when it is demonstrated that the auditor

a. Was negligent.

b. Was grossly negligent.

c. Intended to deceive or harm others.

d. Caused financial loss to an innocent third party.

108. The principal issue to be resolved in cases involving alleged negligence is usually

a. The amount of the damages suffered by the users of the financial statements.

b. Whether to impose punitive damages on defendant.

c. The level of care required to be exercised.

d. Whether defendant was involved in fraud.

109. "Absence of reasonable care that can be expected of a person in a set of circumstances" is the definition

of

a. Ordinary negligence.

b. Constructive fraud.

Page 36: MCQs - prelims

Red Sirug Page 36

c. Gross negligence.

d. Fraud.

110. The limitation of auditor liability under contract law is known as

a. Contributory liability.

b. Privity of contract.

c. Statutory liability,

d. Common law liability.

111. Privity of contract exists between the

a. Auditor and the Securities and Exchange Commission.

b. Auditor and client.

c. Auditor and third parties.

d. All of the above

112. As a consequence of his failure to adhere to generally accepted auditing standards in the course of his

examination of the ABC Corporation, Bugoy, CPA, did not detect the embezzlement of a material amount of

funds by the company's controller. As a matter of common law, to what extent would Bugoy be liable to

Leis Corporation for losses attributable to the theft?

a. He would be liable for losses attributable to his negligence.

b. He would have no liability, since the ordinary examination cannot be relied upon to detect defalcations.

c. He would have no liability because privity of contract is lacking.

d. He would be liable only if it could be proven that he was grossly negligent.

113. A CPA is criminally liable if he

a. Refuses to turn over the schedules or working papers prepared by the client staff to the client.

b. Performs an audit in a negligent manner.

c. Intent ionally allows an omission of a material fact required to be stated in a financial

statement.

d. Was not able to submit the audited financial statements on time.

114. The auditor's legal liability to third parties under common law extends to?

a. All third parties for all acts of negligence.

b. All third parties for acts of willful misconduct.

c. All third parties for fraud; selected third parties for gross and ordinary negligence,

d. All third parties for acts of fraud and gross negligence; selected third parties for ordinary

negligence.

115. The auditor's defense of contributory negligence is most likely to prevail when

a. Third party injury has been minimal.

b. The auditor fails to detect fraud resulting from management override of the control

structure.

c. The client is privately held as contrasted with a public company.

d. Undetected errors have resulted in materially misleading financial statements.

116. A CPA will most likely be negligent when the CPA fails to:

a. Correct errors discovered in the CPA's previously issued audit reports.

b. Detect all of a client's fraudulent activities.

c. Include a negligence disclaimer in the CPA's engagement letter.

d. Warn a client's customers of embezzlement that may be perpetuated by the client's employees.

117. The concept of privity may be important in defending auditors against potential claimants. Privity in

general only allows:

a. Lenders of the client to sue the auditor.

b. Clients to sue their auditors.

c. Anyone that relied upon the audited financial statements to make a decision to sue the auditor as long

as the auditor knew or should have known of such reliance.

d. Shareholders who relied upon the audited financial statements to make an investment decision.

118. Public accountants are not prohibited from providing which of the following services for public audit

clients?

a. The function of the internal controls

b. Selection and implementation of an accounting information system

c. Quarterly financial statement bookkeeping

d. Audits of the effectiveness of internal control

Page 37: MCQs - prelims

Red Sirug Page 37

119. An expectation of the public is that the auditor will recognize that the primary users of audit services are:

a. The employees

b. The investors and creditors

c. The SEC

d. The board of directors

120. What is the primary purpose of effective internal control?

a. Obtaining profitability and financial strength

b. Achievement of certain organizational goals.

c. Completion of a successful audit for the entity

d. Shareholder involvement in the company’s success

121. The overriding objective of the International Auditing Standards that are issued by the International

Auditing Practices Committee of the IFAC is

a. To override a country’s regulations governing the audit of financial statements.

b. To improve the uniformity of audit ing practices and related services throughout the

world.

c. To provide a uniform application of specific audit procedures that are acceptable worldwide.

d. To replace generally accepted auditing standards.

122. Management’s assertions in the financial statements are relevant to the audit process because:

a. They provide evidence that auditors have prepared financial statements in accordance with GAAP

b. They embody the audit procedures that will be performed by the audit engagement team.

c. They include representations of financial statements in accordance with the applicable

reporting criteria

d. They relate to regulator’s expectations about audit results.

123. A CPA firm is considered independent when it performs which of the following services for a publicly

traded audit client?

a. Tax return preparation as approved by the board of directors.

b. Serving as a member of the client’s board of directors.

c. Accounting information system design and implementation.

d. Determining which accounting policies will be adopted by the client.

124. Jessie Garcia, CPA, forgot to test a client’s assessment of goodwill impairment during an audit. Such an

act is probably an example of:

a. Reckless professional behavior

b. Ordinary negligence

c. Due diligence

d. Fraud

125. Similar to auditors in the CPA realm, internal auditors also strive to possess:

a. Independence

b. Competence

c. Objectivity

d. All of the above

126. Analytical procedures are used

a. To set materiality limits.

b. To assess the reasonableness of financial statement amounts.

c. To provide direct evidence about the numbers in the financial statements.

d. To test internal controls.

127. The risk that material misstatements have occurred in transactions entering the accounting system is

a. Audit risk

b. Inherent risk

c. Control risk

d. Detection risk

128. A review engagement differs in scope as compared to an audit due to

a. The subject matter of the audit.

b. The quantity and type of evidence obtained.

c. Ethical requirements with respect to independence.

d. The users of the financial statements.